MBE Crim In-depth

Ace your homework & exams now with Quizwiz!

What are some synonyms for "intent to cause serious bodily harm" murder?

"Great bodily injury" murder, or "grievous bodily injury" murder.

What is the standard of proof for "intent to inflict serious bodily injury murder"?

"Intent to inflict serious bodily injury murder" must be proved by examination of all the surrounding circumstances, including the words and behavior of the defendant. Similarly, the intentional use of any deadly weapon in a way that is likely to cause serious injury provides evidence of intent to inflict serious bodily injury.

What are some means by which the specific "intent to permanently deprive" may be proven, despite no intent on the part of the defendant to keep or to sell the item?

"Intent to permanently deprive" may be proven through proof of intent to keep, destroy or hold property for ransom. It may even be shown by demonstrating that a defendant recklessly exposed the property to loss or dealt with the property in a manner involving a substantial risk of loss.

What are "malum prohibitum" crimes and "malum in se" crimes?

"Malum prohibitum" crimes. These are acts that are wrongful only because they violate a statute (e.g., speeding, failing to register a firearm). "Malum in se" crimes. These are acts that are inherently wrong or "evil;" acts that involve a general criminal intent or moral turpitude (e.g., murder, theft, battery)

Define "blackmail."

"Obtaining the property of another by the use of threats of future harm to the victim or to his property."

What is a synonym for "depraved heart murder?"

"extreme recklessness murder."

What is a common synonym for "voluntary manslaughter?"

"heat-of-passion killing."

What are the successful defenses against "intent to kill" murder?

(absent justification, excuse, or mitigation to voluntary manslaughter)

_____________ is requried for at least _______________ of a strict liability crime.

1. "No particular mental state" 2. "one element"

What is required for one to be an accessory after the fact?

1. A completed felony must have been committed; 2. The accessory must have known of the commission of the felony; and 3. The accessory must have personally given aid to the felon to hinder the felon's apprehension, conviction, or punishment.

What are the elements of the crime of obtaining property by false pretenses?

1. A false representation (beyond puffery) of a present or past material fact (not opinion) by the defendant; 2. that causes the victim to pass title to his property; 3. to the defendant; 4. defendant knows that the representation is false; 5. and thereby intends to defraud the victim.

1. What is a misdemeanor? 2. At common law, what crimes were considered misdemeanors?

1. A misdemeanor is a crime punishable by imprisonment for less than one year or by a fine only. 2. At common law, crimes not considered felonies were deemed misdemeanors.

What amount of force may a police officer use in enforcing the law?

1. A police officer may use that amount of non-deadly force that he reasonably believes necessary to effect a lawful arrest or prevent the escape of the arrestee. 2. A police officer may use deadly force only to prevent the commission of a dangerous felony or to effectuate an arrest (of a person reasonably believed to have committed a felony where it reasonably appears necessary to the officer).

What are the different ways in which the element of actus reus may be met?

1. A voluntary act that causes an unlawful result; 2. an omission to act where the defendant is under a legal duty to act; 3. Vicarious liability where the defendant is responsible for the acts of another party.

Four situations in which a defendant who does not personally commit any acts sufficient to amount to actual cause may nonetheless be legally responsible for a killing.

1. Accomplice liability. 2. Homicide is the reasonably foreseeable result of a conspiracy, and the homicide was committed in furtherance of the conspiracy.. 3. Third party and defendant together cause a victim's death, victim would not have died but for the actions of the third party, and third party's act was neither abnormal nor unforeseeable. 4. Defendant causes the death of another, even if not at his own hands, during the commission of or in an attempt to commit a felony (i.e. felony murder).

In the most general terms, what must a prosecutor prove?

1. Actus reus; 2. mens rea; 3. concurrence in time betweern the act and the requisite mental state; 4. SOME (but not most) crimes require the occurrence of a restult for the crime to be complete (e.g., homicide crimes require that the victim die).

What are the elements of kidnapping?

1. At common law, kidnapping consists of: a. an unlawful b. restraint of a person's liberty c. by force or show of force d. so as to send the victim into another country 2. Under modern law, it suffices that the victim be taken to another location or concealed.

What are the two main ways in which a defendant may commit criminal assault?

1. Attempting to commit battery (majority definition of assault); 2. Intentionally causing the victim to fear an immediate battery.

When may a legal duty to act arise such that failure to act constitutes mens rea? Give an example of each.

1. By statute (e.g., failure to file a tax return); 2. by contract (e.g., failure on the part of a lifeguard, nurse, or guide on a hiking or river rafting expedition to rescue); 3. based upon relationship (e.g., parent/child; spouses) 4. Voluntary undertaking has already begun (e.g., unreasonable abandonment of a rescue that could worsen a victim's plight is sufficient, even if done by a Good Samaritan); 5. Where someone creates a risk of peril to another (If a defendant pushes a victim into a swimming pool as a joke, but then realizes that the victim can't swim when the victim begins to drown, the defendant can be prosecuted for murder for failing to throw the victim a life preserver).

What are the two major defenses to embezzlement?

1. Claim of right. 2. Intent to restore the exact property.

What are the three major defenses to criminal battery?

1. Consent may be a valid defense where it is not coerced or obtained by fraud; 2. Self-defense and defense of others is a valid defense to a battery charge; 3. Where the defendant commits an offensive touching to prevent someone from committing a crime, this will be a defense to battery.

In what situations in which homicide defendants do not personally commit acts sufficient to amount to actual cause of death may the defendants be nonetheless legally responsible for the killing?

1. Defendant who is an ACCOMPLICE to the killer may be held liable for the homicide crime under complicity doctrine. 2. Where the reasonably foreseeable result of a CONSPIRACY is a homicide, and that homicide was committed in furtherance of the conspiracy, then ALL members of the conspiracy may be held liable for the homicide. 3. Both the third party and the defendant together cause the victim's death, and each is a substantial factor in causing the death. 4. FELONY MURDER: Where a defendant causes the death of another, even if not by his own hand, during the commission of a felony. 5. OMISSIONS in the face of a legal duty that result in a death.

What are the four different kinds of malice?

1. Express malice. This is intent-to-kill in the homicide sphere. Deadly weapons doctrine. Intent to kill may be inferred when a defendant uses a deadly weapon against a vulnerable part of a person's body. This latter part is easy to satisfy (can be an arm, as arteries run through arms). 2. Implied malice: Intent to cause serious bodily harm. 3. Implied malice: Depraved heart murder. 4. Implied malice: Felony murder.

1. What is a felony? 2. What were the crimes considered felonies at common law?

1. Felony: A crime punishable by death or by imprisonment for more than one year. 2. Under the common law, burglary, arson, robbery, rape, larceny, murder, manslaughter and mayhem were considered felonies.

Name the five classes of specific intent crimes.

1. First-degree murder; 2. Theft crimes (such as larceny, robbery, extortion, embezzlement, false pretenses and receiving stolen property); 3. Burglary (but not arson, which is a "malice" crime); 4. Inchoate crimes (solicitation, conspiracy and attempt); 5. assault

What are the elements of forgery?

1. Fraudulent making/alteration; 2. of a false writing with apparent legal significance; 3. With the intent to make wrongful use of the forged document.

Abel and Barbara attack Charlie. Abel intends to kill Charlie, but Barbara intends only to seriously injure him. 1. If Charlie survives the attack, can Abel and Barbara be convicted of attempted murder? 2. If Charlie dies, can Abel and Barbara be convicted or Murder?

1. If Charlie does not die, then only Abel can be convicted of attempted murder. Even in a jurisdiction that recognizes murder based on a mens rea, which itself does not require specific intent to kill (e.g. murder based on intent to seriously injure), to be guilty of attempted murder, one still must have a specific intent to kill. 2. If Charlie dies, both Abel and Barbara can be convicted of murder. Abel can be convicted of intent-to-kill murder, and Barbara can be convicted of murder based on intent to seriously injure. Important takeaway. There is sometimes a higher intent requirement for attempt than for the completed offense: To be convicted of murder, one need not have the specific intent to kill. To be convicted of attempted murder, one must have the intent to kill.

What are the different tests for the validity of an insanity defense?

1. M'Naghten Test; 2. Irresistible Impulse Test; 3. Durham/New Hampshire/Product Rule; 4. MPC/Substantial Capacity

1. When does one act "knowingly?" 2. What is the significance of lack of knowledge. 3. What is the relationship between intent and knowledge?

1. One acts "knowingly" when he knows the nature and/or result of his conduct. -One acts "knowingly" with respect to a result when he is substantially certain that his conduct will cause a certain result. 2. Lack of knowledge can often excuse criminal liability under the defense of mistake of fact. 3. Traditionally, intent has been defined to include knowledge.

When can defense-of-property be used as a criminal defense?

1. Reasonable non-deadly force is justified in defending one's property from theft, destruction or trespass where the defendant has a reasonable belief that the property is in immediate danger and no greater force than necessary is used. a. This means that the use of force is improper where a request to desist would suffice. 2. Non-deadly force is also proper when used to re-enter real property or to regain possession of wrongfully-taken personal property upon "immediate pursuit." 3. Deadly force may never be used to merely defend property. a. However, deadly force may be used where the defender reasonably believes an entry will be made or attempted in his dwelling by one intending to commit a felony therein.

What are the three categories of strict liability crimes?

1. Regulatory offenses (e.g. traffic violations, vehicle offenses, administrative statutes); 2.Public welfare offenses (e.g., statutory rape, bigamy). 3. Transferred intent doctrine preserves liability where a defendant intends criminal conduct against one party but instead harms another party, so that his actions bring about an unintended, but still criminal, result.

What are the six classes of defenses?

1. Responsibility (or lack thereof); 2. Justification; 3. Entrapment; 4. Mistake; 5. Consent; 6. Condonation.

What are the major limitations on felony murder?

1. The underlying felony must be collateral. Underlying felony must be independent of the homicide. -A minority of states consider burglary with the intent to commit assault to be noncollateral, and the felony-murder rule is therefore inapplicable to that crime in those jurisdictions. A majority of states, however, find burglary with the intent to commit assault to be a collateral crime to which felony murder doctrine applies if a death results. 2. The felony must be an inherently dangerous one (burglary, arson, robbery, rape, kidnapping); 3. The resulting death must be a foreseeable outgrowth of the defendant's actions. Most courts have generally been very liberal in applying the forrseeability requirement. Most deaths are considered foreseeable for felony murder purposes. 4. The resulting death must occur during the commission or perpetration of the felony. (Res gestae doctrine) 5. Agency rule (vicarious liability, "shooter" rule). Person who committed the killing must be an agent of the felony. At common law, all felons were liable for any homicide that occurred during the perpetration of the felony. It did not make exceptions for homicides caused by non-felons. (e.g., during robbery, a convenience store clerk shoots and kills the accomplice of the robber or a bystander; robber would be found guilty of felony murder.) Under the majority rule today, there is no felony murder liability when a non-felon kills a co-felon or a bystander. In minority jurisdictions, deaths caused by non-felons do count as felony murders. In some minority jurisdictions, felony murder liability will not be imposed if the person killed by the person killed was a felon (Redline limitation).

What are the ways in which a larceny "taking" might be conducted?

1. Through trespass (i.e. without consent); 2. By trickery (i.e., consent to the defendant's taking possession is induced by misrepresentation.)

Defendant attempts to kill Victim by tying him up and pushing him into the pool. At what point in this sequence of events could the Defendant be considered to have attempted murder: 1. under the common law? 2. under modern attempt doctrine?

1. Under the common law "last act" doctrine for attempt, defendant likely would have had to push the tied-up Victim into the pool to be guilty of attempted murder. 2. Today, tying up the Victim next to the pool without pushing him in would likely be sufficient to constitute attempt under any of the modern doctrines of attempt, to include "proximity," "equivocality," "dangerous proximity" and "substantial step."

1. Describe the Pinkerton doctrine. 2. What is its current status in American jurisprudence?

1. Where a conspirator does not have the sufficient mens rea for liability as an accomplice, the Pinkerton doctrine provides that each conspirator is liable for the crimes of all the other co-conspirators where the crimes were both a. a foreseeable outgrowth of the conspiracy and b. committed in furtherance of the conspiratorial goal. 2. Most jurisdictions have rejected the Pinkerton doctrine and will not impose liability upon a conspirator for crimes committed in by the other co-conspirators merely upon the basis of the agreement.

What are the elements of common law burglary?

1. the breaking (heavily relaxed) 2. and entering 3. of the dwelling house (largely eliminated) 4. of another 5. in the nighttime (largely eliminated) 6. with the intent to commit a felony therein

Define embezzlement.

1. the fraudulent conversion or misappropriation; 2. of the property of another; 3. by one who is already in lawful possession.

What are the elements of the crime of receiving stolen property?

1. the receiving of stolen property; 2. known to be stolen; 3. with the intent to permanently deprive the owner.

What are the elements of larceny?

1. the taking 2. and carrying away ("asporation") 3. of the tangible personal property 4. of another 5. with the intent to permanently deprive the owner thereof

Define "general intent crime."

A "general intent crime" is a crime that merely requires commission of an unlawful act (e.g., nonconsensual intercourse), without a specific mens rea. A generally-bad state of mind will suffice where such a criminal act is committed voluntarily and purposely. Rule of thumb. This is usually satisfied by a reckless or negligent state of mind.

A private citizen mistakenly kills a non-felon who the citizen believed to be a felon (either through mistaken identity, or because no crime in fact was committed). When may such a citizen assert the defense of mistake? When is such a defense impermissible?

A citizen who kills a non-felon in such circumstances will be able to assert mistake if 1. the mistake was made in good faith and was made reasonably. 2. the reason for the use of force was self-defense or to prevent the commission of a dangerous felony. A private citizen defendant who mistakenly uses deadly force to prevent the escape of a fleeing felon is not justified, however, if no dangerous felony has actually been committed, or if the victim was not actually the felon.

Define "specific intent crime." What are the two methods of possessing specific intent.

A crime that involves more than the objective fault required by merely doing the proscribed actus reus. A defendant will possess specific intent if: 1. He wants, hopes, or wishes that his conduct will bring about a particular result, regardless of the objective likelihood of the result occurring (unless the result is inherently impossible); or 2. He expects (i.e. is substantially certain) that his purposeful act will have that particular result, even though he does not necessarily want a particular result. Rule of thumb. Purpose or knowledge as to the result will suffice. Example: Joe wants to kill his cousin Mike, so he tosses a brick off of the roof of the 50-story building where Mike works at noon, knowing that Mike is usually walking somewhere for lunch at that time. Despite the long odds, the brick actually hits mike and kills him. Joe has the requisite specific intent to be charged with Mike's murder.

Under what situations could one be found liable for either/both larceny or/and embezzlement?

A defendant could never be found liable for both or either. Unlike larceny, embezzlement involves misappropriation by a defendant who has lawful possession (as opposed to custody)

What is the presumption regarding sanity?

A defendant is presumed sane until he raises evidence as to his sanity.

What is the mens rea requirement for criminal battery?

A defendant may be guilty of battery where he acts: recklessly, negligently or with the knowledge that his act or omission will result in criminal liability.

When may a person being arrested use force to resist the arrest?

A defendant may use reasonable, non-deadly force to resist an unlawful arrest. A minority of jurisdictions do not permit even non-deadly force in order to resist an unlawful arrest. Most jurisdictions permit an individual to use force to resist an arrest made with excessive force, especially if the individual fears injury. An individual may only resist a lawful arrest by a police officer where the individual does not know that the other person is a police officer.

George D. Painter is strolling past Manx's Cycle when he sees a red FXR that is identical to the one that he currently owns in every way (down to the mileage), save one thing. The one in the shop has the same serial number as his first motorcycle. He decides to switch his current bike for his long-lost love. He does this in a clever way. He asks the sales guy at Manx's if he can go for a "test drive." He drives the bike to his home, picks up his current bike, and rides the latter bike to Manx's, intending to pretend that it's the one that he just took for a test drive. During the ride back, he is stopped for a traffic violation, the serial number of George's bike is written down, and George is arrested for embezzlement, as Manx's got suspicious of George and called the cops. When questioned about why George is returning to Manx's witha substantially identical bike with a different VIN, George spills the beans. In his defense, he argues that he returned Manx's an identical bike to the one that twas equivalent. What result?

A defendant who converts property but intends to later substitute it for equivalent property is still guilty of embezzlement.

A bank robber robs a bank with a REAL gun. As he's running out of the bank with the plastic gun, he sees a cop with weapon drawn upon him. The robber SHOOTS AT THE COP IN AN ATTEMPT TO KILL HIM before the cop can fire. The cop shoots back twice, both of which rounds land in an old lady's forehead, killing her instantly. Can the robber be charged with felony murder? What about the getaway driver; can he be charged?

A defendant who starts a gun battle or an an exchange of fire with police, victims or bystanders will be deemed to have engaged in wanton and willful misconduct. Such a defendant could be held liable for murder for any resulting homicides without applying the felony-murder rule. Any co-felons of the defendant would also be responsible for the wanton and willful misconduct-murder under general principles of accomplice liability. Note that this also applies to those who take hostages as shields. A minority of jurisdictions now make an exception to felony murder liability for a nonviolent co-felon who was not armed and did not participate in or have any knowledge of the violent co-felon's intentions (was told by the bank robber that his guns were fake).

Chuck goes to a fraternity party and meets Diane. They hit it off and go to Chuck's room, where they bang. The next morning, Diane's father and the cops are at his door. Chuck is arrested for rape, as it turns out that Diane is 15. Chuck asserts that he was mistaken as to Diane's age, given the context of the situation. What result?

A defendant's mistake as to the age of the victim is generally no defense to statutory rape.

Define "larceny by trick."

A form of larceny whereby the defendant obtains possession of the personal property of another by means of a representation or promise that he knows is false at the time he takes possession.

Define "homicide."

A killing of a human being caused by another human being (the defendant). More specifically, an action or actions of the defendant that cause the death of another human being with criminal intent and without legal excuse or justification.

Define "felony murder."

A killing proximately caused during the commission or attempted commission of a serious or inherently dangerous felony. Includes both intentional and accidental killings. Includes liability for the acts of one's accomplices.

What is a lesser-included offense? What are the lesser-included offenses of robbery?

A lesser-included offense is an offense where, if the offense occurs, all of the elements of the lesser offense are included within the greater offense. The lesser-included offenses of robbery are larceny, assault and battery.

A 15-year-old girl is leaving her high school, when she sees a UPS man that she fancies. She goes up to him and says: "will you agree to have sex with me?" He says yes. She then says: "OK, then, meet me in front of the school tonight." The Vice Principal has overheard the exchange and tells the girl's parents to make sure she doesn't leave the house tonight. The Vice Principal then notifies the cops, who wait for UPS guy in front of the school that evening. When he shows up, the cops arrest him. He is then charged with conspiracy to commit statutory rape and no other crime. What should be his defense?

A member of a legislatively protected class cannot be guilty of conspiracy to commit that crime or of being an accessory to that crime.

Define "solicitation" under the common law. Difference between common law and MPC definition of socicitation?

A misdemeanor including enticing, advising, inciting, inducing, urging, or otherwise encouraging another to commit a felony or breach of the peace. MPC definition includes requesting another to commit any offense (including misdemeanors and felonies).

When talking first-degree murder, what is a "deliberate" murder?

A murder performed with a cool mind. This refers to the defendant's ability to coolly reflect.

A agreed to lend his car to B so that B could rob a bank. A tells B that he changed his mind. What else must A do to cut off accomplice liability in the event that B goes ahead with the robbery. What need he NOT do?

A needs to get his car keys back from B to cut off accomplice liability in the event that B goes ahead with the robbery. He need not, however, call the police and tell them of B's plan.

How much non-deadly force is a private citizen privileged to use in enforcing the law?

A private citizen is privileged to use that amount of non-deadly force that reasonably appears necessary: 1. to prevent the commission of a felony or a misdemeanor amounting to a breach of the peace, or 2. to make an arrest if the crime was a. committed in fact and b. the defendant reasonably believes the person against whom he uses the force committed the crime.

When is a private citizen privileged to use deadly force to enforce the law?

A private citizen may use the same amount of deadly force as a police officer only if a dangerous felony is involved and the person against whom he used the force is actually guilty of the crime.

Describe the defense of public duty.

A public officer, policeman, or private citizen offering assistance is justified in using reasonable force against another or in taking the property of another, provided that he is acting within his authority pursuant to a law, court order, or process that is valid or that he reasonably believes to be valid.

What is "aggravated assault?"

A simple assault may rise to the level of an aggravated assault under certain circumstances. Most commonly, the circumstances include: 1. Where the defendant commits an assault with a dangerous weapon, or 2. where the defendant acts with the intent to seriously injure, rape or murder the victim.

A 15 year old girl goes up to a 21 year old frat boy and tells him: "have sex with me." He does, and is later arrested for statutory rape. She is also arrested, for solicitation, as she encouraged the frat boy to commit statutory rape. What is the girl's bulletproof defense?

A solicitor may not be guilty of the offense of solicitation if she is a member of the class of persons the law seeks to protect.

What is the level of voluntariness required for the actus reus of murder?

A voluntary act, an involuntary act arising from a voluntary act (e.g., person who has frequent seizures driving a car), or an omission to act where there is a legal duty to act.

A flashes B a "thumbs-up" sign just before B beats C. B would have beaten C anyway. Is A's act sufficient for complicity?

A's act is sufficient for complicity if B saw it and felt even slightly encouraged.

Abe takes a swing at Bob. Bob responds by shooting Abe with a gun. Abe grabs a knife and stabs Bob in the throat. Bob dies. Abe is tried for murder and raises the defense of self-defense. The prosecution responds to this defense by arguing that Abe was required to retreat, as Bob was simply exercising his right of self-defense when he fired the shot. Result?

Abe's defense is valid. Abe was legally justified in stabbing Bob, since Bob escalated force.

What kinds of mental disorders do not count as a "mental disease or defect" for the purposes of MPC insanity? Give an example.

Abnormalities manifested only by repeated criminal or otherwise antisocial conduct, such as Kleptomania

Which physical acts performed by a defendant which result in harm to others NOT be considered actus reus?

Acts that are reflexive, convulsive, performed while unconscious, or otherwise involuntary. Such acts are considered mere bad thoughts accompanied by action. Habitual acts of which one is simply unaware are, however, considered conscious and voluntary. (e.g., a chain smoker who lights a cig in a no-smoking area w/o realizing it can be successfully prosecuted.) Acts which are performed under duress are also VOLUNTARY.

What is the standard for the "adequate provocation" required for "voluntary manslaughter" to have taken place?

Adequate provocation must be such that a reasonable person would lose self-control.

In what factual situations will courts generally find "adequate provocation" to reduce a defendant's criminal liability from murder to voluntary manslaughter? In what factual situations will courts generally NOT find such "adequate provocation?"

Adequate provocation present: -Courts will commonly find that a defendant was adequately provoked when he killed after he was the victim of a serious battery or a threat of deadly force or where he found his spouse engaged in sexual conduct with another person. -In some jurisdictions, imperfect self-defense may mitigate murder to voluntary manslaughter where a defendant was either at fault in starting an altercation, or unreasonably, but honestly, believed that harm was imminent or that deadly force was necessary. Such mistaken justification has been applied to self-defense, defense of others, crime prevention, coercion and necessity. -A minority of states allow diminished mental capacity short of insanity to reduce murder to manslaughter. Adequate provocation not present: Where a defendant kills after an exchange of "mere words," courts will generally not find adequate provocation.

What constitutes a "disease of the mind" under the M'Naghten Test?

All mental abnormalities, but not a psychopathic personality.

To what extent are accessories after the fact punished?

An accessory after the fact is not an accomplice, nor is he punished to the same extent as prescribed for the parties to the felony. Modern statutes largely focus on punishing accessories after the fact under the separate crime of "obstruction of justice," and not by holding that "accessory after the fact" vicariously liable for the underlying felony.

Can an accomplice be found responsible for crimes other than the crime assisted and encouraged?

An accomplice is responsible not just for the crime assisted and encouraged, but also for other crimes that are the natural and probable consequences of the crime assisted or encouraged. The MPC limits accomplice liability to only the crime assisted or encouraged.

Define aggressor.

An aggressor is one who strikes the first blow or commits a crime against the victim.

Define "conspiracy."

An agreement between two or more persons to commit a crime.

Define "voluntary manslaughter."

An intentional killing mitigated by adequate provocation or other circumstances negating malice aforethought.

Define "involuntary manslaughter."

An unintentional killing resulting without malice aforethought caused either by recklessness, (criminal negligence/gross negligence) or during the commission or attempted commission of an unlawful act that fall short of felony murder. The majority of jurisdictions do not require that the defendant be consciously aware of the risk created.

What is a "depraved heart murder?"

An unintentional killing that results from conduct involving a wanton indifference to human life and a conscious disregard of an unreasonable risk of death or serious bodily injury.

Define "second-degree murder." Give examples

Any murder that does not meet the requisite elements of first-degree murder. Examples: defendant's malice is intent to inflict serious bodily injury; where the defendant acted with wanton and willful misconduct; or felony murder, where the underlying felony is not specifically listed in an applicable first-degree murder statute.

Today, what is the mens rea for murder under modern common law?

Any of the following suffice under the modern common law: 1. Intent to kill; 2. Intent to cause serious bodily harm; 3. Depraved heart murder; and 4. Felony murder

What are the two malice crimes?

Arson and common law murder.

What were the common law rules regarding the defense of infancy? What is the current state of these rules?

At common law, a complete defense due to incapacity existed for children under 7 years of age. Children between the ages of 7-14 were rebuttably presumed to lack criminal capacity. Children over age 14 were held responsible as adults. Many states have abolished the common law presumptions and established a specific minimum age required for a criminal conviction.

What was the common law rule regarding causation and the amount of time between the defendant's act and the victim's death? What is its current status?

At common law, if the victim died more than one year and one day after the defendant's act, the courts would rule that the defendant's act was not the proximate cause of the killing. Most states have either eliminated this rule or have extended the period within which the defendant is held legally responsible.

At common law what was the mens rea for murder?

At common law, it was malice.

How old must one be to commit rape?

At common law, males under 14 were conclusively presumed incapable of rape. many modern jurisdictions maintain the presumption but make it rebuttable.

Define the crime of "mayhem."

At common law, mayhem required an intent to maim or do bodily injury, accompanied by an act that either: 1. Dismembered the victim 2. Disfigured the victim, or 3. Disabled his use of some bodily part that was useful in fighting. Modern law has expanded the scope of mayhem to include 3. permanent disfigurement

What are "infamous crimes?"

At common law, these were crimes that involved fraud or dishonesty.

The counsel of a corporation holds a meeting with the board. He proposes to the board a brilliant Ponzi scheme in the guise of a new stock offering. The board agrees to go with the scheme. While the counsel is on the phone with the advertising department of Business Week, arranging for an ad to be placed for the "issue" of this "new stock," the cops are listening in and recording. The counsel is arrested and charged with conspiracy with the corporation to commit embezzlement. What defense should the counsel use?

Because a corporation and its agent are considered one person, no conspiracy can exist between them.

A defendant who is being prosecuted under the felony murder doctrine has an "extra layer" of defense. What do I mean by this?

Because the defendant must be guilty of the underlying felony, any defense to the underlying felony negates the felony murder.

What is a synonym for extortion?

Blackmail.

Defendant shoots Victim in the shoulder causing a serious but not life-threatening injury. Shortly thereafter, Third Party stabs Victim repeatedly in both feet, causing numerous bleeding wounds. Hours later, Victim dies from excessive blood loss. both the shoulder and feet woulds caused the bleeding. Who, if anyone, is criminally liable, and for what homicide crime? Explain?

Both will be found guilty of murder. Where both a third party and a defendant together cause a victim's death, the victim would not have died but for the actions of the third party, and the third party's act was neither abnormal nor unforeseeable, the actions of both the defendant and the third party will be considered direct causes of the death.

Franny Frugel was walking home from work when Mugger attacked her, shooting her in the leg. As Franny struggled to find her cell phone, Batterer hit her in the head with a brick, knocking her unconscious. Franny, now unable to move and thus unable to call for help, bled to death from the leg wound. Is Mugger liable for her death? Is Batterer?

Both will be held criminally responsible for Franny's death. Both will be held to have caused her death for the purposes of criminal law. With respect to cause-in-fact, we have the substantial factor test to deal with multiple parties. In this case, each perpetrator was a substantial factor in Frannie's death, so you would have cause in fact. You'd also have proximate cause, so both are guilty.

What constitutes "tangible personal property" capable of being the subject of larceny?

Common law larceny was limited to tangible personal property. Modern statutes have expanded the kinds of property subject to larceny to services and other intangibles (such as ripping off the gas and electrical companies power and stealing written instruments that represent property rights).

Define "intent to kill," as that term is used to describe one of the means of satisfying the mens rea requirement of murder.

Conduct where the defendant consciously desires to kill another person or makes the resulting death inevitable.

One of the elements of the crime of embezzlement is "fraudulent conversion or misappropriation." What is "conversion?" When is conversion not fraudulent?

Conversion is some action toward property such as selling, consuming, pledging, donating, discarding, heavily damaging or claiming title to it, which *seriously interferes with the rights of the owner*. Slight movement or limited use, for instance, would not suffice, but use that deprives the owner of a significant portion of the usefulness of the object would be sufficient. Conversion is not fraudulent when the defendant honestly believed he had a right to use the property as he did.

Define "battery" in the criminal sense.

Criminal battery is the intentional, reckless, or criminally negligent unlawful application of force to the person of the victim. Must be an "offensive touching," and the touching must be unlawful (e.g., security guard can physically stop a shoplifter lawfully)

What conditions must be present for liability to be imposed on a defendant for an omission to act?

Criminal liablity can be imposed on a defendant for an omission to act where: 1. There is a legal duty to act; and 2. The defendant can physically perform the act.

What does criminal negligence usually require? Distinctions and similarities between criminal negligence and recklessness?

Criminal negligence usually requires that the defendant's conduct create a high degree of risk of death or serious injury beyond the tort standard of ordinary negligence. The degree of that risk, however, has no precise, objective measure. It is more than mere ordinary negligence but less than wanton and willful misconduct. Criminally negligent behavior requires more than just ordinary negligence. It requires *gross* negligence. Like criminal negligence, it involves a substantial and unjustifiable risk that is a gross deviation from the standard of care expected of a person in a similar situation. The main difference between recklessness and criminal negligence is that the latter means that you *should have been aware of the risk,* whereas reckless means that you *disregarded* the risk.

Define "deadly force" and explain when it may be used in self-defense.

Deadly force: Force which treatens death or serious bodily harm. Deadly force can only be used in self-defense in response to an attack that threatens death or great bodily injury.

Defendant drives his car over Victim's legs, intending to break them. However, Victim ends up dying as a result of the injuries suffered. Could Defendant be prosecuted for murder?

Defendant has the necessary mens rea for murder. The intentional use of any deadly weapon in a way that is likely to cause serious injury provides evidence of intent to inflict serious bodily injury.

In crim law, what are the tests used to determine whether a defendant's actions were the "actual cause" of a criminal result?

Defendant's actions were the "actual cause" of a criminal result if either: 1. the criminal result would not have occurred BUT FOR defendant's act; OR 2. there are multiple causes or other parties responsible for the criminal result, AND DEFENDANT'S ACT WAS A SUBSTANTIAL FACTOR IN CAUSING THE CRIMINAL RESULT.

For crimes which require that the occurrence of the crime be complete (e.g., murder requiring a dead victim), there yet more are additional requirements. What are they?

Defendant's conduct must be both the actual cause and the proximate cause of the specified criminal result.

Describe the concurrence requirement of criminal law.

Defendant's criminal intent must occur at the time he commits the criminal act. Moreover, the mental state must put into action the criminal act or omission.

A pro football player is walking down the street, clutching his man-purse. A slight woman walks up to him and points a very realistic-looking toy gun upon him, telling him that she'll shoot him to death unless he gives up the purse. The football player is extremely frightened for his life. For that reason alone, he hands the purse to her. A cop witnesses this, and arrests the woman for robbery. At trial, the defendant argues that there was no robbery, as the victim's rear was baseless. Result?

Defense will fail. If a victim is placed in fear, though baseless, there may still be a robbery. Fear based on lies qualifies as threats for constituting robbery.

Santino is hanging out in front of his warehouse, a solid, thick-walled, concrete building with thick, locking, solid-steel doors. Santino sees a car coming down the road toward him. He recognizes the car as belonging to a man who has threatened his life, and sees that there is another man brandishing a Tommy gun standing on one of the running boards of the car. Santino understands that an attempt will be made on his life. rather than step into his fortress-like building and close the heavy steel door behind him, Santino reaches into his jacket and pulls out a .45 pistol, takes aim, and shoots the driver dead. The car runs into a building, killing the gunman. Santino is tried for murder. At trial, he claims self-defense, to which the prosecution responds by arguing that Santino had the opportunity to escape being harmed in any way by simply taking one step, shutting a door, and calling the cops which would have prevented two deaths. What result?

Depends on the jurisdiction. Majority rule: Santino's defense is valid. the general rule is that a person who did not initiate conflict has no duty to retreat in the face of a deadly attack. Minority rule: A number of jurisdictions DO require retreat if it is feasible and can be done in safety before using deadly force. These jurisdictions: -do not require retreat if it cannot be done safely or if the defendant is in his own home, auto or workplace. Even if in one of those places, however, a defendant in a retreat jurisdiction must retreat before using deadly force if the defendant is the aggressor. -are split as to whether one must retreat in one's own dwelling if attacked by a co-dweller.

For no reason, Greg pulls a knife and attacks Peter. Peter struggles with Greg and succeeds in disarming Greg. Just as Peter was obtaining control of the knife, Bobby walks around the corner, sees Greg and reasonably thinks that Greg was about to be stabbed by Peter. Bobby pulls a pistol and shoots and kills Peter. At trial, Bobby asserts the defense of defense-of-others. What result?

Depends on the jurisdiction. Majority rule: The majority rule focuses on the reasonableness of the defendant's belief that the third person was being unlawfully attacked. In a majority jurisdiction, Bobby would be excused. The minority rule is that the defendant has no more right to use deadly force than the third-person ostensibly being protected. In a minority jurisdiction, Bobby would therefore not be excused.

Can one who begins to attempt a crime, but later abandons his attempt, raise the defense of abandonment if he is prosecuted for attempt?

Depends. At common law, abandonment is not a defense to attempt once the attempt is complete. The obvious reason for this is that the common law requires that the "last act" necessary to achieve the intended result be performed for there to be an attempt. Obviously, by then, it is too late to withdraw. Many jurisdictions, to include MPC jurisdictions, recognize a *voluntary* and *complete* abandonment as a defense. -Voluntary: A true change of heart, not simply giving up in the face of difficulties or an increased likelihood of being caught. -Complete: The defendant is not merely postponing commission of the crime.

With what crime can the survivor of a Russian roulette game be charged?

Depraved heart murder. An unintentional killing that results from conduct involving a wanton indifference to human life and a conscious disregard of an unreasonable risk of death or serious bodily injury. There is a substantial risk of death.

Describe the "collaterality" limitation on felony murder doctrine, and its practical consequences.

Description: The underlying felony must be independent of the homicide so that every felonious attack upon a victim that is ultimately fatal does not become escalated to murder by the rule. Consequence: A prosecutor cannot charge a defendant with felony murder on the sole basis of an assault because the assaultive conduct is not independent of the resulting homicide. However, a prosecutor could charge felony murder against a kidnapper who accidentally allowed his captive to starve to death, because the kidnapping would be considered independent of the resulting homicide.

Distinguish direct cause and indirect cause by defining them.

Direct cause. When teh defendant's actions alone cause the harm, the defendant's act is the DIRECT CAUSE, and it is very likely that the defendant will be held legally responsible. Indirect cause. In indirect cause situations, the other force that combines with the defendant's act to bring about the harm is called in intervening cause. An intervening cause. If an intervening cause breaks the chain of causation, it is a superseding intervening cause.

How much assistance or encouragement is sufficient to meet the actus reus requirement of complicity?

Even slight assistance or encouragement is sufficient. However, the defendant must actually assist or encourage. The assistance or encouragement need not be a cause in fact of the commission of the crime.

When is the "carrying away" or "asporation" element of larceny complete.

Even the slightest movement will suffice, even six inches.

A drunken car nut sees a classic car rear ended by a taxi. He sees the drivers of both cars get out of their vehicles and exchange words. The car nut yells out "shoot that cabbie with a musket!" A cop arrests the car nut for solicitation. At trial, the car nut raises the defense of impossibility, as--in retrospect--it is clear that the driver of the old car, who was wearing shorts and a tee shirt, was not armed with a musket. Result?

Even where it would be impossible for the solicitee to carry out the crime, such impossibility is no defense to the crime of solicitation. What matters is what the solicitor believes the circumstances to be, not what they are.

What is the prerequisite of an insanity defense under all of the theories under which it may be brought?

Evidence of mental illness or history.

What are the different kinds of ex-post-facto laws?

Ex post facto laws are those laws that retroactively: 1. make conduct criminal; 2. enforce stricter punishment for the same conduct; or 3. alter procedural or evidentiary rules in such a way that the criminal defendant may be more easily convicted.

Abel is a crackhead. His friend Bob, who is from another state, is visiting Abel one day. Bob sees that Abel has a ****** car. Abel notices Bob's dismay at seeing the state of Abel's car, so Abel asks to borrow a couple thousand from Bob to buy a new car. Bob lends the money, which Abel takes. Abel absconds with the money, which he spends on crack. Of what is Abel guilty?

False pretenses. If, in response to teh defendant's false representations, teh victim lends money or grants title, teh defendant is guilty of false pretenses.d

What is "first degree murder?"

First-degree murder includes intent-to-kill murder committed with premeditation and deliberation, felony murder, and--in some jurisdictions--murder accomplished by lying in wait, poison, terrorism or torture. Some jurisdictions require little or nothing more than an intent to kill in order to find premeditation and deliberation, but most jurisdictions require more. Most jurisdictions require a reasonable period of time for premeditation and some evidence of reflection in order to distinguish first-degree murders from "spur-of-the-moment" killings.

Describe the M'Naghten Test.

Focuses on the defendant's reasoning abilities. A defendant is relieved of criminal responsibility upon proof that, at the time of commission of the act, he was laboring under such a defect of reason from a disease of the mind as not to know the nature and quality of the act he was doing or, if he did know it, not to know that what he was doing was wrong.

How does one determine whether a defendant's actions were the "actual cause" of a victim's death?

For common law murder, the "but for" test applies. The fact finder must determine that the victim's death would not have occurred but for the defendant's actions. Even if the defendant's actions alone would be insufficient to cause the victim's death, but in fact contributed to the death, a court may still find actual causation.

What amount of force is required for robbery by force?

Force need be only more than that which is required to move the property.

What is meant by non-deadly force?

Force which threatens only non-serious bodily harm.

Is subsequent forgiveness by the victim ever a defense to the commission of a crime?

Generally, no.

Is it a defense that the criminal defendant didn't know that his acts were illegal?

Generally, no. Ignorance of the law is not a defense. There are some exceptions, however: 1. Where a statute proscribing the defendant's conduct has not been reasonably made available or where the defendant has reasonably relied on a statute or judicial decision that is later overruled or declared unconstitutional; 2. Where a defendant relies in good faith upon an erroneous official statement of law contained in an administrative order or in an official interpretation by a public officer or department; and 3. Where some element of a crime involves knowledge or awareness of the law by the defendant.

What is the general rule regarding defense of others?

Generally, one is justified in using force to defend a third person to the same extent that the person using force would be justified in using force to defend themselves. The majority rule focuses on the reasonableness of the defendant's belief that the third person was being unlawfully attacked.

What are the limits to the entrapment defense (i.e., when and how may the government encourage criminal activity)?

Government officials may encourage criminal activity by providing the opportunity or the equipment for the commission of crime.

Under what circumstances may an accomplice cease to be liable for any future crimes committed by the person or people with whom he is complicit?

He must both withdraw and abandon: 1. He must give no further assistance or encouragement; 2. He mus communicate his withdrawal to his accomplices. 3. Many jurisdictions require him to make efforts to neutralize his prior assistance or encouragement, but he does not have to try to thwart the commission of the crime.

A pilot fails to do a walk-around prior to take-off. He consequently does not notice that there is a hole in the plane's rudder. This results in him having no ability to yaw the plane at landing. He eventually catches a tailwind that lines the plane up with a highway. He has a chance to land on the highway now, which will likely kill or injure at least one person. If he does try to land on the highway now, he will likely not have another chance to land again. Pilot lands, killing a motorist. He is tried for manslaughter. He raises the necessity defense. What result?

His necessity defense will not fly (no pun intended). there is no defense of necessity where the defendant is at fault in creating the perilous situation. Had he done a walk-around, as he should have, he would have seen the hole in the rudder and not flown.

Neighbor #1 complains to me that his lawnmower was stolen. The next day, I see Neighbor #2 mowing his lawn with Neighbor #1's mower. I comment to Neighbor #2: "hey, that's a nice mower." He offers to sell it to me for $50. I take his offer, with the intent to give it back to Neighbor #1, but before I do so, I notice my lawn needs mowing. I feel that I've earned the use of the mower, since I've helped its owner. I start mowing my lawn, at which point, the cops arrest me for receiving stolen property. In my defense at trial, I state that I intended to return the mower. Result?

I will win. The defendant must, at the time he receives the stolen property, have the specific intent to permanently deprive the owner of that property. Where a defendant intends, at the time of receipt,l to unconditionally return property to its owner, tahjt defendant is not guilty of receiving stolen property.

Can a person who was intoxicated, but who was nonetheless was sober enough to be able to form the intent to kill, be able to use an intoxication defense to avoid liability for first-degree murder? If not, why not? If so, how?

If a defendant was voluntarily intoxicated, but nonetheless sober enough to be able to form the intent to kill, he may be able to avoid liability for first-degree murder if he can prove that the intoxication precluded him from acting with premeditation or deliberation.

As a bank robber flees on foot from the scene of the robbery, he enters a crowded street. He wishes to clear his route of egress from any potential "heroes" who might try to get in his way and stop him, so he fires a bunch of shots in the air. One of the shots ricochets off of an overhanging street light and kills a man. The defendant is prosecuted for murder under the felony murder doctrine. He defends himself under the theory that, under the felony murder doctrine, the death must occur during the commission or perpetration of the felony, while in his case the death occurred during flight from the scene. What result?

If a killing occurs while the defendant is fleeing from the scene of the felony, he may still be guilty of felony murder. However, if he has reached a place of temporary safety, the felony is deemed to have terminated, and the defendant can no longer be found guilty of felony murder.

Describe the defendant's defense of self-defense.

If a person has a reasonable belief that he is in imminent danger of unlawful bodily harm, he may use that amount of force in self-defense that is reasonably necessary to prevent such harm, unless he is the initial aggressor. Two rules of thumb: -The force must be proportional to the initial attack; -The initial attack must be wrongful.

Under the prevailing view, when is voluntary intoxication a valid defense for a specific intent crime?

If it negates the requisite mental state.

In what situation will a defendant who has created a very high risk of death or injury and thereby unintentionally killed another be acquitted of depraved heart murder? In other words, what is the unique defense to depraved heart murder?

If the defendant created the risk for a socially reasonable purpose.

Bill broke into Steve's home and took a computer. He left the computer in his yard, where it got rusted and dirty. One year later, Bill felt guilty and returned the computer to Steve. Is Bill guilty of larceny? OR Describe the Morgan Freeman "can of peas" defense.

If the defendant takes property with the intent to pay for it or later replace it, such intent may negate the intent to permanently deprive element if, for example, the property is easily replaceable.

All day, Spongebob and Patrick have been tempted by the balloons on display in Toys-R-Us. While hanging out in the mall, they hatch a plan whereby one of them will distract the only salesperson working at T-R-U, while the other guy steals a balloon. A police officer in the mall overhears this exchange and follows them, sure enough, they are about to walk into the store, when the cop arrests them. they are both charged with conspiracy. They later learn that it was "free balloon day" at TRU, and therefore raise impossibility as their defense. What result?

Impossibility is not a defense to conspiracy.

Give an example where an accomplice may be found liable for crimes other than the crime assisted and encouraged.

In common law jurisdictions, an accomplice to an armed bank robbery can be held responsible for the death of a bank teller who is shot by another defendant during the bank robbery.

Main difference between larceny by trick and false pretenses?

In larceny by trick, the defendant's fraud is used to cause the victim to convey possession, not title, as in false pretenses.

What is aggravated battery?

In most jurisdictions, certain circumstances cause a simple battery to be elevated to an aggravated battery. Most commonly, these circumstances include: 1. The defendant causing the victim serious bodily injury; 2. The defendant using a deadly weapon to commit the battery; or 3. The defendant battering a woman, child or law enforcement officer.

What is the "incompetency" defense, and how does it differ from the defense of insanity?

Incompetency refers to a doctrine that prevents defendants from being tried, convicted or punished unless they have the sufficient present ability to consult counsel with a reasonable and rational understanding of the proceedings. If a defendant is found incompetent, he can later be tried and punished if the competency is restored. Insanity concerns the defendant's mental state at the time the offense is committed. Competency is assessed at any time during the pendency of the criminal case in court.

What are the defenses that deal with the defendant's lack of responsibility?

Insanity, Competency, Diminished Capacity, Intoxication, Infancy.

What is the mens rea for solicitation?

Intent that the solicitee perform criminal acts.

What mens rea is required for assault in a majority of jurisdictions?

Intent, knowledge, purposefulness, or willfulness. Since an intent to injure is required in such jurisdictions, recklessness or negligence that comes close to causing injury will not suffice for an assault.

Under what circumstances is involuntary intoxication a valid defense to criminal charges?

Involuntary intoxication is a defense to a crime, even if it does not negate an element of the crime, under the same circumstances as insanity.

I have Ballsack. I want to piss him off and make him feel some pain. I go up to him with a stiff sheet of paper, and inflict a paper cut upon him. Little do I know, he's a hemopheliac. He bleeds out and dies. What is the highest crime of which I may be charged?

Involuntary manslaughter. The attempt to injure is, by definition, either a battery or an assault. these are both malum in se crimes. Therefore, application of the misdemeanor-manslaughter rule renders any resulting homicide an involuntary manslaughter. Remember, there is an "egg-shell skull" rule to homicide, too. The tort maxim "you take the plaintiff as you find him" applies to crime victims as well. That is, if the harm results from a special sensitivity (such as hemophilia or other preexisting medical condition), the defendant's act is a proximate cause of the harm, regardless of whether the defendant could have foreseen the unique medical condition.

What crime results when a defendant intends to injure a victim to a lesser degree but this unexpectedly causes the death of the victim? What is the legal explanation for this result?

Involuntary manslaughter. Legal explanation: The attempt to injure is, by definition, either a battery or an assault. these are both malum in se crimes. Therefore, application of the misdemeanor-manslaughter rule renders any resulting homicide an involuntary manslaughter.

What is the legal origin of the crime of embezzlement?

It is a statutory crime.

What is the origin of the crime of obtaining property by false pretenses?

It is a statutory crime.

Neighbor #1 complains to me that his lawnmower was stolen. The next day, I see Neighbor #2 mowing his lawn with Neighbor #1's mower. I comment to Neighbor #2: "hey, that's a nice mower." He offers to sell it to me for $50. I take his offer, with the intent to give it back to Neighbor #1, but before I do so, I notice my lawn needs mowing. I go to Neighbor #1's home with the mower. I show it to him and say: "check it out; I got the mower. Some guy was selling it, and I bought it off him." Neighbor #1 is overjoyed. I then ask him: "I do expect you to reimburse me the $50 I paid for it, of course." Neighbor #1 refuses to pay and I huff off in anger with teh mower. I am arrested for receiging stolen property. Wahtt result?

It is likely that I will lose. After one receives stolen property, if he conditions return, such as by payment of a reward, this may be evidence of intent to permanently deprive. the defendant does not have to be acting for personal gain.

What level of burning is required to satisfy the "burning" element of arson?

It is not necessary that the dwelling be substantially damaged. Although a mere blackening of the surface is not enough, there must have been some charring of the premises. Mere burning of the furniture or other contents of the dwelling without fire damage to the structure itself is not arson.

The police have intercepted several communications in which Tom has contacted people who have agreed to rob a bank with him. In the last of these communications, Tom tells his would-be co-conspirators: "I'm going to case the joint and report back to you guys the number and locations of entrances and exits." While casing the joint, Tom is arrested. He is charged with conspiracy. The cops could never track down the people with whom he has been communicating. He raises in his defense the argument that there cannot be only one conspirator; a conspiracy requires multiple conspirators. What result?

It is not necessary to try more than one conspirator. Thus, a single conspirator could be tried and convicted of conspiracy if his co-conspirators are missing or dead.

What is the "deadly weapons doctrine?"

It is the doctrine which states that an inference of intent to kill is raised through the intentional use of any instrument which, judging from its manner of use, is calculated to produce death or serious bodily injury. Example. A defendant's intent to kill can be inferred from deliberately swinging a baseball bat at the victim's head.

In which of the different regimes of insanity defenses is insanity easiest to establish?

It is typically easier to establish MPC insanity because it does not require the total or absolute loss of cognitive ability or volition, only that the defendant lacked substantial capacity in tehse areas. It also combines a cognitive component (not knowing) with a volitional component (not controlling) and allows a verdict of insanity for either one.

A car nut sees a classic car rear ended by a taxi. He sees the drivers of both cars get out of their vehicles and exchange words. The car nut yells out "kick that jerkoff cabbie's ass!" The cabbie and the driver of the old car clearly haven't heard the exhortation directed to their situation and politely continue to exchange insurance information. A cop arrests the car nut for solicitation. At trial, the car nut raises the defense that his exhortations clearly were not heard. Result?

It is unnecessary that the person solicited enter into an agreement to commit the requested crime for a solicitation to occur. Indeed, a person solicited to do a crime may not even respond, but the solicitor will still be guilty of solicitation.

In a jurisdiction in which assault is defined as "intentionally causing the victim to fear an immediate battery," Galooly, a hitman with a sniper rifle, attempts to kneecap his girlfriend's figure skating rival from far away. He gets a good sight picture and squeezes the trigger. However, the rifle malfunctions, and the round does not fire. By the time he fixes the problem with his rifle, his target is long gone. As he climbs down the tree, he is arrested by a cop. He is charged with assault. As a defense, he argues that there was no fear or apprehension, and therefore no assault. Will this defense work?

It very well might. In "fear of battery" assault jurisdictions, the fear/apprehension is a key element. No assault has been committed where the victim is unaware of the threat of harm.

A number of states define certain types of kidnapping as being of the aggravated type and thus deserving of higher punishment. Give some examples.

Kidnapper restrains a small child; Kidnapper is masked; kidnapper holds victim for ransom; kidnapper kidnaps for the purpose of committing a robbery or sexual offense.

Abel pays for a car with a worthless check. He runs off with the car. Of what is he guilty?

Larceny by trick, and NOT false pretenses. A defendant who pays for property with a worthless check is guilty of larceny by trick, since title does not pass until the check is cashed.

Of what crime is a person who cheats at cards guilty?

Larceny by trick, and NOT false pretenses. Where money is exchanged by cheating at cards or cheating at other gambling, the defrauding party is receiving possession, not title, so teh crime is larceny by trick, and NOT false pretenses.

A sleazy librarian convinces kids that borrow from the library and their parents that they need to give him $200 before borrowing books "as a security deposit." Of course, there is no need for such a deposit. Of what is the sleazy librarian guilty?

Larceny by trick. When a victim pledges money for security, bails money for safekeeping or gives money to be used only for a specific purpose, and the victim takes such action as a result of the defendant's misrepresentations, the defendant acquires only possession and is guilty of larceny by trick.

Abel is a crackhead. His friend Bob, who is from another state, is visiting Abel one day. While watching a movie with Abel, Bob mentions that he is hungry. Abel is having one of his crack cravings, and tells Bob that he can get a pizza, but that a pizza in his neighborhood costs $36, and that Abel will need $18 from Bob. Abel takes the money, goes to Faz's Pizza, gets a $5 pie and spends the rest of the money on crack. Of what is Abel guilty?

Larceny by trick. When a victim pledges money for security, bails money for safekeeping or gives money to be used only for a specific purpose, and the victim takes such action as a result of the defendant's misrepresentations, the defendant acquires only possession and is guilty of larceny by trick.

Dwayne lives in a ****** part of town, and--for tax reasons--his employer pays him in cash only. Dwayne's friend tells Dwayne that he has turned his basement into a steel-reinforced bank-style vault with a digital combo lock. Because Dwayne doesn't trust banks and does trust his friend, he gives his friend the portion of his pay that he wishes to save. The friend of course has no vault, and absconds with the money. Of what is the sleazy friend guilty?

Larceny by trick. When a victim pledges money for security, bails money for safekeeping or gives money to be used only for a specific purpose, and the victim takes such action as a result of the defendant's misrepresentations, the defendant acquires only possession and is guilty of larceny by trick.

When is an individual criminally liable as an accomplice?

Majority: An individual is criminally liable as an accomplice if he: 1. (actus reus) gives assistance or encouragement or fails to act where he has a legal duty to oppose the crime of another, and he 2. (mens rea) purposefully intends to effectuate commission of the crime. There are two parts to this: -Must intend to commit the acts of assistance or encouragement and -must further intend to encourage or assist another to commit the crime charged. Minority of modern statutes: Accomplice liability is creates when one knowingly assists or encourages a crime, such as in cases where a seller, knowing of the buyer's intent to commit arson, sells him an explosive device.

What are the elements of the crime of attempt?

Mens rea: a specific intent to bring about a criminal result; and Actus reus: a significant overt act in furtherance of that intent.

Jimmy is a bank robber. He wants his little brother Charlie to get into the bank robbery business. He brings Charlie along to a robbery as a sort of "bring your little brother to work day." During the robbery, Charlie stands idle, watching Jimmy do his work. Both Jimmy and Charlie are arrested during the robbery. Charlie is charged with robbery under a complicity theory. What is Charlie's best defense?

Mere presence does not make one an accomplice.

Give some examples of activities that may be deemed criminal negligence.

Mishandling of loaded weapons. Dangerous operation of a motor vehicle, including driving while intoxicated.

Joe is at a party, when Jane starts coming on to him strong. Finally, she says "I'm so tired, I think it's about time for me to go home." Joe takes this as a signal to offer her a ride home, and to bone down. He offers her the ride, which she accepts. After he pulls in front of her place, he starts making out with her. During the make-out session, she goes limp, and he engages in coitus with her. After the quick session, he notices that the reason she went limp is that she's passed out. Her breaths are shallow and her pulse weak. He goes to the emergency room with her, where the medical personnel notice signs of OD and of intercourse. They call the cops on Joe, who arrest him and charge him for rape. Can Joe assert mistake as a defense?

Most courts do not recognize a mistake as to whether the victim consented as a defense, even if the mistake were reasonable. However, consent is determined objectively from the observable circumstances. A victim's subjective lack of consent will not suffice if the objective circumstances suggest that consent existed.

In a majority of jurisdictions, assault consists of an attempt to commit battery. In such a jurisdiction, a defendant, with the intent to shoot the victim, pulls the trigger. Unbeknownst to the defendant, there were no bullets in the weapon. Can the defendant avoid liability for assault if he can prove that he did not have the ability to consummate the battery?

Most states applying the majority definition of criminal assault do not permit a defendant to avoid liability for assault simply because the defendant lacked the present ability to consummate the battery.. In the minority of states, there is an additional requirement, imposed by statute or case law, that the defendant have to present ability to commit the battery.

What mental state is required for a finding of guilt for a general intent crime?

Negligence or recklessness is a sufficient mental state for general intent crimes. While mere ordinary negligence (defined, as in tort, as the failure to use due care) does not amount to criminal negligence; negligence that causes a greater risk of harm than ordinary negligence or ordinary negligence where a defendant is consciously aware of the risk will amount to criminal negligence. Criminal negligence may also be called "gross negligence" or culpable negligence.

A state's embezzlement statute defines the crime as 1. the fraudulent conversion or misappropriation; 2. of the property of another; 3. "by one to whom the property has been entrusted." In this jurisdiction: Abel, a single man living alone, received a package at his door with no name for the addressee. he opened it up, and found a valuable woman's mink coat, which he sells for a nice sum. Bob is a co-owner of a boat with Tom. One day, Bob takes the boat out, and moors it at a secret harbor that he's built. He then fakes an shipwreck. After the USCG rescues him, he tells Tom that he's sorry, but the boat has been wrecked. Tom collects on the insurance and splits it with Bob. Which of these men, if any, can be prosecuted for embezzlement?

Neither Abel nor Bob may be prosecuted for embezzlement. In jurisdictions with embezzlement statutes which define embezzlement as requiring that the property be "entrusted" to the defendant, these statutes have been construed as not applicable to defendants who come into lawful possession of property by finding it or by mistaken delivery (i.e., the property was not "entrusted" to the defendant because it was never intended that he be given possession ). Fraudulent conversion by a co-owner of property is not embezzlement.

When is mistake of fact a defense to a strict liability crime?

Never. Strict liability crimes require no mental state. This is why mistake of the victim's age is not a defense to statutory rape.

A murder defendant peals "not guilty" at arraignment. He later decides to raise an insanity defense. Does the earlier "not guilty" plea waive his right to raise the insanity defense?

No.

Is physical coercion required for compelled intercourse to constitute rape?

No. Intercourse accomplished by mere threats rather than by force alone may constitute rape. Also, if the victim is incapable of consenting (due to intoxication or unconsciousness), the intercourse is rape. Finally, where the female is under the statutorily prescribed age of consent (usually 16), an act of intercourse constitutes rape regardless of her apparent consent.

A suspect has ridden away the motorcycle of another, but leaves a note explaining that he needed to borrow the bike for fifteen minutes, so that he may file some court papers on time. When he gets to court and tries to file the papers, the filing clerk has a million questions, failure to answer which could result in his being found to have filed late. The result is that it takes him three hours to file the papers. He returns the bike late, at which time there are cops waiting for him. Was the leaving of the note be sufficient to preclude the suspect from being charged with larceny?

No. Though it is true that there is no intent to permanently deprive if the defendant intends to return the property to the victim unconditionally and within a reasonable time at the time of the taking, the defendant must have the ability to return the property, even if something unanticipated stops the actual return of the property.

In a jurisdiction in which assault is defined as "intentionally causing the victim to fear an immediate battery," a naked, empty-handed man runs up to the victim, and says "I'll shoot you if you don't give me your money." Has an assault occurred?

No. A defendant who is guilty of assault in a "fear of battery" assault jurisdiction must intend to either cause a reasonable apprehension of an immediate battery or cause the victim to suffer bodily harm. When a reasonable person would not expect imminent bodily harm, there is no criminal assault. Words alone, for example, are generally held insufficient to constitute an "apprehension" assault. Here there is no reasonable apprehension of a battery. The man claimed he would shoot, but he clearly had no gun.

A barber finds a wallet filled with cash, but without ID, laying on the coffee table in his shop. He keeps it behind the register, in the expectation that someone will return to claim it. After six months without anyone claiming the thing, he takes the cash for himself, and throws the wallet itself in a Salvation Army collection box. Is the barber guilty of larceny?

No. Abandoned property cannot be the subject of larceny. A wallet with no identifying marks found in a busy location that has been held in good-faith expectation of search by the owner for six months can surely be said to be abandoned.

Can one embezzle services?

No. Embezzlement deals with tangible personal property, not services. Some modern statutes do include real estate, however.

In a majority of jurisdictions, assault consists of an attempt to commit battery. If the victim was not aware of the attempted battery, is this a defense to this type of assault?

No. Even if the victim was not aware of the attempted battery, an assault still results.

Is consent of the victim a defense to a crime.

No. Except when it negates a specific element of the offense, such as in rape or kidnapping.

An accountant for a corporation that produces asbestos and cigarettes has decided that he's morally against what he has been doing all these years. He doesn't want to lose his job, so he decides to misappropriate company funds each year by donating them to the Lung Association and the Cancer Society in amounts sufficient to defray the costs which the company imposes. Accountant is arrested for embezzlement. He raises in his defense that he did not stand to personally gain from the misappropriation. Will this defense work?

No. For a defendant to be found guilty of embezzlement, no direct personal gain need result to the defendant.

Eric the Midget is coming after me with a pocketknife as I am helping Carmiche unload baseball bats from a minivan. Once I notice that Eric is trying to stab me to death, I pull my gun from its concealed holster and shoot him. Eric dies. When I am tried for a homicide crime, may I use the defense of self-defense?

No. If non-deadly force would stop a deadly attack, responding with deadly force is not reasonable. Here, I had the opportunity to respond effectively with non-deadly force (my attempted killer was a midget and I had a baseball bat in hand).

A pro football player is walking down the street, clutching his man-purse. A slight woman walks up to him and tells him that she'll choke him to death unless he gives up the purse. The football player is extremely conflict averse. For that reason alone, he hands the purse to her. A cop witnesses this, and arrests the woman for robbery. Will the charges stick?

No. If robbery is based on the threat of violence, the threat must place the victim in actual fear at the time of the taking.

In a jurisdiction in which assault is defined as "intentionally causing the victim to fear an immediate battery," Dickbag comes up to my car and threatens to punch me through the window and shows me his fist. He then punches through the window and into my face. Can he be charged with both assault (for the threat) and with battery (for the physical striking)?

No. If the feared battery is accomplished, the assault and battery merge, and the defendant is found guilty of only the battery.

I am a skilled mime. I want to scare an enemy. I take a very realistic fake sledgehammer made of Styrofoam and pantomime attacking my enemy with it, so that the enemy will be scared witless and convinced that he is being attacked, but so that no injury will result if, despite my attempts to prevent contact, the fake hammer nonetheless actually makes contact. I am arrested after sending my enemy running scared. In a majority jurisdiction, can I be prosecuted for assault?

No. In a majority of jurisdictions, assault consists of an attempt to commit battery. In such jurisdictions, an intent merely to frighten, though accompanied by a fear-producing act, will not suffice for a conviction for assault.

Jerry walked into the ice cream shop and told Ben that he'd just banged Ben's new girlfriend. Ben was outraged and suddenly decided to beat Jerry to death. He began pounding on Jerry's head with an ice cream scoop. Jerry died as a result of the beating. What degree of murder did Ben commit in a majority of jurisdictions?

No. In a minority jurisdiction, Ben may have committed first-degree murder, but not in a majority jurisdiction.

If a defendant's attempt at committing a crime is successful, can the defendant still be prosecuted for attempt?

No. Once the target crime is committed, the attempt merges into the target crime.

We're at the shooting range. I put a gun to Helio's head and tell him to shoot Scott or I will kill him. Helio does so. He is later charged for murder Can he raise a duress defense?

No. There is no duress defense for murder.

Tom and Vito are bank robbers. While chatting in a public park, Tom and Vito agree to rob a particular bank. They later rob the bank. They are charged with both conspiracy and robbery. Tom is ready to concede to the robbery charge, but feels that--since the robbery was completed--the additional conspiracy charge is redundant. Is he correct?

No. Unlike attempt, conspiracy is a separate and distinct offense that does not merge upon completion of the target crime because criminal combinations are deemed to be dangerous apart form the underlying crime itself.

I accidentally take someone else's umbrella upon leaving a restaurant. Am I guilty of larceny?

No. Where a defendant, at the time of teh taking, has a good-faith belief that he si entitled to possession, there is no intent to permanently deprive, even if that belief is both incorrect and unreasonable.

I'm a skilled sniper. I hide in a tree two miles outside of my enemy's house and wait for him to leave in the morning. When he steps out, I take my time to get a good bead on him. As I get ready to pull the trigger, a bolt of lightning strikes him, killing him instantly. I had my eyes closed during the lightning strike. I open them and get ready to pull the trigger. Right before the thunder cracks, I pull the trigger and put a bullet in him as he's falling to the ground. can I be found guilty of murder?

No. Where an intervening act occurs that is outside the universe of foreseeable events caused by the defendant's acts, such an act will cut off the chain of causation, and the defendant will be acquitted of murder. Additionally, a dead person cannot be killed. Thus if an intervening cause kills the victim before the defendant can complete his act, he will be acquitted.

Boone, a 21-year-old, legally marries Anna Nicole, a 14-year-old. Can he be prosecuted for rape for the sex they had on their wedding night?

No. Where the parites are validly married, a husband cannot be convicted of "statutory" rape.

Carl thinks his neighbor Ralphie is weird. One day he walks by Ralphie's apartment and notices that the place seems to be empty and quiet, and that the door is cracked open. Out of curiosity, Carl pushes the door fully open, walks in, and sits down on the couch. He looks at the coffee table and sees a Sportster engine case being used as a paperweight. Realizing that Ralph isn't using the case for its full potential, Carl picks it up, and walks off with it. Is Carl guilty of burglary? explain.

No; Carl isn't guilty of burglary. To engage in burglary, one must do the breaking and entering WITH THE INTENT to commit a larceny. Here, this was not the intent of Carl's breaking and entering. Thus, there is no burglary. In short, the concurrence requirement is not met for the breaking and entering and the intent.

During a bank robbery, a bank employee spills coffee on her keyboard and electrocutes herself. Can the robbers be found guilty of murder under felony murder doctrine?

No; the fact that a homicide occurs simultaneously to and in the same location as a dangerous felony will not, by itself, make the felony-murder rule applicable. The defendant's actions must still be causally connected to the victim's death.

Sleazy P. Martini convinces me in bad faith that, contrary to fact, my Shelby GT500 is not an authentic one, but a replica. I sell it to him at a bargain basement price. While showing an old Shelby tech some photos of my "repilca" one day, he recognizes my car, and tells me that --not only was it a real Shelby--but it was a one-of-a-kind. I get pissed and call the cops. Sleazy is charged and convicted with obtaining property by false pretenses. After the conviction, I talk to Sleazy and his family, and they convince me that Sleazy had a real love for that car. My heartstrings are pulled, and I decide that Sleazy should keep the car. What do I have to do to give Sleazy effective title?

Nothing. When the crime of obtaining property by false pretenses takes place, title passes to the defendant, even though it is voidable due to the defendant's fraud.

When does one act "intentionally?"

One acts "intentionally" when he desires that his acts cause certain consequences or when he knows that his acts are substantially certain to produce those consequences. Rule of thumb: Usually includes "purposely" or "knowingly." Usually one acts with intent towards a result if he acts with purpose or knowledge.

When does one act "recklessly?"

One acts "recklessly" with respect to a material element of an offense when he consciously disregards a substantial and unjustifiable risk that the material element exists or will result from his conduct. The risk must be of such a nature and degree that, considering the nature and purpose of the actor's conduct and the circumstances known to him, its disregard involves a gross deviation from the standard of conduct that a law-abiding person would observe in the actor's situation. Breakdown: 1. Conscious disregard; 2. Substantial risk; 3. An unjustifiable risk; 4. That is a gross deviation of the standard of care required of a person in a similar situation.

When does one act "purposely?"

One acts purposely when there exits a conscious objective to engage in certain conduct or to cause a certain result.

When does one act willfully?

One acts willfully when she acts both "intentionally" and "purposely." "Wilful" has also often been used to imply that a crime involved evil purpose in moral turpitude crimes. That is, when she both: 1. desires that her acts cause certain consequences knows that her acts are substantially certain to produce those consequences, 2. acts with the conscious objective to engage in such conduct or to cause such a result, and 3. (sometimes) has acted with evil purpose in crimes involving moral turpitude.

Define "accessory before the fact." To what extent may an accessory before the fact be punished?

One who aids, abets, counsels or otherwise encourages the commission of a felony, but is not present at the scene is guilty as an accessory before the fact. To what extent may an accessory before the fact be punished? An accessory before the fact may be punished to the same extent as a principal for all crimes committed within the scope of the conspiracy.

Under the old common law, what was a "principal in the second degree?" To what extent may a principal in the second degree have been punished for his participation in the crime?

One who is present at the scene of the felony and aids, abets or otherwise encourages the commission of the crime with the requisite intent. For example, a getaway car driver who waits outside during a bank robbery is guilty as a principal in the second degree. A principal in the second degree may be punished to the same extent as the perpetrator. Note that modern law has abolished teh distinction between principals in the first and second degree.

When can a death resulting from a "malum prohibitum" crime be sufficient for involuntary manslaughter?

Only when the killing is either a foreseeable consequence of the unlawful conduct, or amounts to criminal negligence.

Proof of what physical act is required to prove rape?

Penile penetration of a vagina, regardless of how slight. Other nonconsensual sexual conduct is covered under separate crimes.

What are the positive requirements and the negative requirements for the "adequate provocation" requirement of "voluntary manslaughter?"

Positive requirements: -Legally adequate provocation (limited at common law to witnessing spouse in act of adultery, watching family member subjected to violent/sexual assault, mutual combat, being personally subjected to violent assault). Though some states have since expanded these categories, mere words are still not enough. -Actual provocation is required. A causal connection must exist between the legally-adequate grounds for provocation and the killing; Negative requirements: -The time period between the heat-of-passion and the fatal act must not be long enough that a reasonable person would have cooled off. -Actual cooling-off must not have taken place. Mitigating circumstances reducing a defendant's criminal liability from murder to voluntary manslaughter will not be found where a defendant actually did cool off (even if a reasonable person would not have cooled off) or a defendant who, for any other reason, killed while they were subjectively not in the heat of passion.

Under the old common law, in criminal cases involving an accomplice, what was the term for the actual perpetrator(s) who performed the criminal act with the requisite mental state?

Principal(s) in the first degree. Note that modern law has abolished teh distinction between principals in the first and second degree.

1. Which mental states may be negated by voluntary intoxication? 2. Which mental states may not be negated by a voluntary intoxication defense, and for what crimes may voluntary intoxication NOT serve as an excuse?

Purposeful and knowing mental states. It may therefore be a valid defense for specific intent crimes when it negated the requisite mental state. Voluntary intoxication is NOT a defense to general intent crimes and will not negate recklessness, negligence or strict liability.

What are the general intent crimes?

Rape, battery, kidnapping, false imprisonment, involuntary manslaughter and depraved heart murder.

Describe the "necessity" defense.

Reasonable force is justified to avoid imminent injury resulting from natural (non-human) forces (e.g. pilot is legally justified in crash landing his disabled plane on a highway, knowing it will kill several motorists) or where an individual reasonably believes that his criminal conduct is necessary to avoid a "greater harm" that would result from compliance with the law. (e.g. Samaritan kills Gunman to save a schoolhouse of Children) This defense also protects military personnel acting within their duties and public executioners.

What level of force may be legally used in self-defense?

Reasonable force. this is the amount of force that is necessary to avoid the threatened harm.

What action on the part of the victim is sufficient to prove rape in a majority of jurisdictions?

Resistance. In some jurisdictions the simple absence of affirmative consent is sufficient.

Define "robbery."

Robbery is a larceny, plus two additional elements: 1. The taking must be from the person or presence of the victim (meaning an area within his control), and 2. by force or violence, or 3. intimidation or the threat of violence -

What are the different justification defenses?

Self defense, defense of a third person, defense of property, necessity, and law enforcement.

Can mental defect short of insanity be a defense to criminal charges?

Some jurisdictions allow the defense of diminished capacity, which is short of insanity, to prove that, as a result of a mental defect, the defendant did or did not have a state of mind that is an element of the offense. The diminished capacity defense is used to negate a specific mental state required for the particular crime.

Describe the Durham/New Hampshire/Product Rule.

States that a defendant is not criminally responsible if his unlawful act was the product of mental disease or defect (meaning that it would not have been committed "but for" the defect or the disease).

When a defendant asserts that a mistake of fact negates the existence of general intent, what must he prove?

That the mistake of fact was reasonable to the extent that under the circumstances a reasonable person would have made that type of mistake.

When a defendant asserts that a mistake of fact negates the existence of special intent, what must he prove?

That the mistake was honest. It need not have been reasonable.

What are the elements of arson?

The 1. malicious 2. burning 3. of the dwelling (has been expanded in most states to include virtually any structure) 4. of another (has been expanded to include burning one's own structures for purposes of fraud, as with insurance fraud.).

Describe and name the test that used under the MPC to determine the validity of an insanity defense.

The MPC uses the "substantial capacity" test. Under the "substantial capacity" test, a person is not responsible for criminal conduct if, at the time of such conduct, as a result of mental disease or defect, he lacked substantial capacity to appreciate the criminality (wrongfulness) of his conduct or to conform his conduct to the requirements of law.

What is the causation requirement for murder?

The act must actually and proximately cause the death of another living person. The common law requirement for a living person was a person "born alive," though a state may extend criminal liability to include a fetus after the first trimester. The death must be caused by a person OTHER THAN THE VICTIM. To persuade or aid another to commit suicide is a sufficient basis for murder in some jurisdictions, though. The defendant's conduct must be both the actual cause and a legal cause of the victim's death.

At common law, what was rape?

The act of unlawful sexual intercourse by a male person with a female person without her consent. Though a man could not rape his wife at common law, this is no longer the law in any state.

A guy comes up to me and offers to sell me a gold top LP, which he tells me he stole from Les Paul's estate. I give him the money, and instruct him to leave the guitar, in its case, in a designated hotel lobby. I am arrested and tried for receiving stolen property before I can pick the guitar up. I defend myself by arguing that I cannot be said to have met the element of "receiving" the property, as I was not given possession of it. Result?

The actual "receiving" of the property is not required, so long as the defendant exercises control over the goods (e.g., arranging for sale, having the thief place the goods in a designated place, or receiving the goods from another.

What is the "actus reus" of conspiracy?

The agreement itself. A majority of jurisdictions now also requires an overt act in furtherance of the conspiracy.

What is the difference between premeditation and deliberation in the murder context?

The defendant deliberated if (he/she) carefully weighed the considerations for and against (his/her) choice and, knowing the consequences, decided to kill. The defendant premeditated if (he/she) decided to kill before acting.

A hitman knows that his prey goes to bed at 10:30 every night. He formulates a plan: at 11:00, riddle the bedroom with machine gun fire. In his experience, and with his machinegunning skills, he knows that doing so will kill anything in that bedroom. The hitman carries out his plan, but is arrested as he flees the scene. It is discovered that that particular night, his prey did not come home from work, but spent the night at his girlfriend's house. The hitman is charged with attempt. He raises the defense that he did not really attempt to kill anyone, since nobody was home. Result?

The defense will fail. Factual impossibility is not a defense to attempt.

A burglary defendant breaks a window with a hammer, because he wants to enter that window to steal the hope diamond, which he knows is located in the building. After breaking the window, he notices that the hole in the glass is surrounded by shards which he will be unable to remove, and which would cut him if he attempts to enter dressed as he is. He returns home and gets a thick leather jacket and jeans to protect him. He enters the window and starts looking for the diamond. Before he can find it, the cops arrest him. He is charged and tried for burglary. He raises the defense that there was no "breaking and entry," but rather a separate "breaking" and a separate "entry." Result?

The defense will fail. The breaking and entering are sufficiently related, regardless of their distance in time from one another.

A car nut sees a classic car rear ended by a taxi. He sees the drivers of both cars get out of their vehicles and exchange words. The car nut yells out "kick that cabbie's ass!" The cabbie and the driver of the old car shake their heads disapprovingly at the car nut, exchange insurance information and drive off. A cop arrests the car nut for solicitation. At trial, the car nut raises the defense that no assault or battery took place, as he suggested. Result?

The defense will fail. The offense of solicitation is complete at teh time teh solicitation is made. Completion of the offense solicited is unnecessary.

Tom has been hired as the "motor-pool operator" for an upcoming bank robbery. He buys a group of cars that will suffice as transportation for the robbers and money, tests them for soundness, and prepares them for the robbery, then instructs each driver in their use. All of the actual robbers and the ringleader are arrested after the robbery. They are tried, but are all acquitted. Finally, the cops learn of Tom's role in the robbery, and have solid evidence that he assisted in the robbery. They arrest Tom and charge him with robbery on complicity grounds. In his defense, Tom raises the argument that he cannot have been an accomplice to a robbery of which all the supposed robbers were acquitted. What result?

The defense will fail. Under the majority view, the principal need not be convicted in order for the accessory before the fact to be convicted. At common law, conviction of teh principal was required for conviction of an accessory.

Eric Estrada wants his cousin's farm, so he makes a bullshit pitch to the cousin. He tells the cousin that the farm is in an area that is going to be irredeemably flooded within a five-year period. Eric even shows his cousin some convincing-looking hydrographic charts showing this. The cousin was already tired of farming and was thinking about giving the farm to a family member; Eric's notifying him of the impending disaster to befall his farm made him firmly decide that he not only needed to give the farm away to a family member, but that Eric was the guy to give it to, since Eric claimed that he had a plan to save the farm. He tells Eric all of this in an e-mail. Eric's cousin passes title to Eric, then finds out later that Eric signed some lucrative mineral rights to the land. Eric is arrested and charged with obtaining property (his cousin's money) by false pretenses. In defense, Eric introduces the e-mail from his cousin, to negate the element that the cousin's reliance on the representation must cause him to pass title. The e-mail indicates that dissatisfaction with farming, and not just reliance on the misrepresentation is what caused the cousin to pass title. What result?

The defense will fail. While the victim's reliance on the representation must cause him to pass title, it does not need to be the only reason that the victim passes title.

I walk past Cafe Racer and see their beautiful Silver Jubilee Bonneville. I can't help myself. The keys are in the ignition, and I ride away, vowing never to return the bike to her unappreciative owners. Then I feel guilty for being a thief. I return the bike, but there are cops waiting for me when I do so. I am charged with larceny. In court, I raise the defense that I could not be guilty of larceny, as I could not be said to have "intended to permanently deprive" Cafe Racer of the bike. Moreover, I explain that the people at CR forgave me for my theft of their bike. What result?

The defense will not likely succeed. Intent is not conclusively established by proof that a defendant actually returned the property to the victim. If the taking occurred with the requisite intent to permanently deprive, a defendant will not be relieved of liability for larceny because the defendant later returns the property or if the victim later forgives the defendant.

In a jurisdiction in which assault is defined as "intentionally causing the victim to fear an immediate battery," Salvatore Giunta is approached by a man with a gun who threatens his life. Sal has faced gunfire before, and is not scared. Sal acts calmly, pulls some kung fu moves, takes the guy down and executes a citizen's arrest. The defendant is charged with assault. The defendant argues that he committed no assault, since he didn't cause any fear, as Giunta didn't feel any fear. What result?

The defense will not work to exonerate the defendant. The element of "apprehension" in this type of assault connotes "expectation more than "fear," even when the term "fear" is employed in the assault crime statute at hand. The victim does not have to actually be afraid but rather must simply and reasonably anticipate or expect that the defendant's acts will result in immediate bodily harm.

I want to prank my neighbor. I set off a prank "bomb" in his garage, which consists of a fireworks show within the garage the fuse for which is lit when my neighbor enters the garage. He enters the garage and the fireworks start going off. Some gas cans near the car (for the lawn mower) catch flame and set the house on fire. I am charged with arson. As a defense, I argue that I did not have intent to burn the dwelling, and that arson requires malice. Result?

The defense will not work. The defendant need not intend to burn the dwelling. Malice requires only that the defendant intentionally take an action that involves a substantial risk of burning.

I walk past Cafe Racer and see their yellow Ducati. I can't help myself. The keys are in the ignition, and I ride away. I have a plan, though. I'm wearing gloves, so no fingerprints. Moreover, I'm not going to keep it. I'm just going to ride the **** out of it, then dump it somewhere, where the cops will eventually find it and return it to the owners. This is exactly what happens. However, a few kids recognized my unique helmet, and the cops are able to identify me as the thief. I am charged with larceny. As a defense I raise the fact that I could not be said to have "intended to permanently deprive." What result?

The defense won't work. The intent element will be satisfied where a defendant intends, when taking the property, to recklessly use it temporarily, then abandon it, hoping someone will return it to the victim, even if the property somehow does return to the victim.

On what does the difference between the various homicide crimes typically depend?

The difference between the various homicide crimes typically depends upon the mental state of the defendant with respect to the conduct causing the death.

Spongebob sees a gaggle of beautiful helium balloons floating on the ceiling of a Toys-R-Us store, their long ribbons hanging almost to the floor. Spongebob is tempted to take one. He sneaks up to the balloons, grabs one of the ribbons, and runs off as fast as he can. A cop stops him, however, and arrests him. Spongebob is charged with larceny. He calls his lawyer, who informs him that it was "free balloon day" at Toys-R-Us. What is the name of the doctrine on which Spongebob will rely?

The doctrine is legal impossibility, and is valid.

Under the felony murder rule, the death must occur during the commission or perpetration of the felony. For the purposes of felony murder, when does the felony start?

The felony starts when the defendant could be convicted of attempting the underlying felony. There is no requirement that the felony be completed. Ends when the perpetrator has arrived at a place of safety.

All day, Spongebob and Patrick have been tempted by the balloons on display in Toys-R-Us. While hanging out in the mall, they hatch a plan whereby one of them will distract the only salesperson working at T-R-U, while the other guy steals a balloon. A police officer in the mall overhears this exchange and follows them; sure enough, they walk up to the threshold of the store. Spongebob and Patrick then look at each other, wince, shake their heads, and walk away from the store. The cop then arrests them. They are tried for conspiracy. The two men raise in their defense that they withdrew from their plan. Result?

The general rule is that withdrawal is not recognized as a valid defense to conspiracy because the conspiracy is complete as soon as the parties agreed to commit the crime and an overt act is committed.

What is the specific intent required for larceny?

The intent to permanently deprive. this must accompany the taking. This may be demonstrated by an intent to keep, destroy, or hold property for ransom.

What is the mens rea of conspiracy?

The mental state required is both: 1. Intent to agree, and 2. Intent to achieve the objective of the agreement.

What is the modern trend regarding consent in the law of rape?

The modern trend is to require actual consent of the victim. If there is no consent, the sexual act is unlawful.

When may parents of a minor child use force upon the child? Where else is the parents' defense in this situation (parent-child) available?

The parents of a minor child, or one in loco parentis, may justifiably use reasonable force on the child to promote the child's welfare. Reasonableness is determined in light of the child's age, sex, health, and particular misconduct based on the totality of the circumstances. This defense is available in some other situations where similar responsibility lies, such as a ship's captain for his crew, or a warden for his prisoners.

When is the requisite intent for malice met?

The requisite intent for malice is met when a defendant acts intentionally or with reckless disregard of an obvious or known risk that the particular harmful result will occur.

When a finding of proximate cause is found in crim law, when is proximate cause present?

The resultant harm is within the risk created by the defendant's conduct in crimes involving negligence or recklessness, or sufficiently similar to that intended in crimes requiring intent, so as not to hold the defendant liable for extraordinary results (such as acts of nature or grossly negligent or intentional bad acts of third parties, including intentional medical mistreatment).

What is the test for determining whether the "overt act in furtherance of the criminal result" is sufficient to constitute an attempt?

The significant overt act must be beyond an act of mere preparation. -At common law, the defendant was required to have performed the "last act" necessary to achieve the intended result. -today, acts prior to the last act are usually sufficient. --Many courts apply a "proximity" test, which asks how close in time and physical distance a defendant was to the time and place the target crime was to be committed. For crimes involving danger to human life, many courts apply a "dangerous proximity" test. --Some courts use an "equivocality" test, which requires that the defendant's conduct be such that it can have no other purpose than the commission of the crime attempted. --Jurisdictions following the MPC require only an act that constitutes a "substantial step" toward the offense3 that corroborates the criminal intent required, such has scouting the scene of the intended crime.

What is the timing requirement regarding the "taking," the "carrying away" and the "intent to permanently deprive" elements of larceny?

The taking and the carrying away must concur in time with the intent to permanently deprive.

What defenses apply to specific intent crimes?

The typical criminal defenses apply to specific intent crimes. Voluntary intoxication and unreasonable mistake may negate requisite specific intent elements.

What is the common law definition of "murder?"

The unlawful killing of a human being with malice aforethought.

Defendant planned to kill his wife. He returned home late. Defendant drove into the driveway and noticed that the entire house was surrounded by snow drifts. Defendant entered the home and pulled a gun on his wife. The wife escapes and runs outside. Dressed only in a nightgown, she hides in the doghouse rather than seek help in the cold. She dies of exposure. Can defendant be found to have proximately caused the Victim's death?

The wife's death by exposure after hiding in the doghouse is an independent intervening cause, but was foreseeable. So, defendant's act might be sufficient to impose criminal liability, even though his wife's reaction was an independent intervening cause.

Davey the drug dealer meets up with Antoine, and the two men develop a friendship. Davey tells Antoine about a great plan that he has to import heroin into the U.S, using the caskets of fallen U.S. servicemembers as carrying cases for the drugs. He explain that the last piece of the puzzle is getting the heroin out of the caskets after the planes touch down at Dover AFB. Davey tells Antoine that, if Antoine is in, he'll start investigating how to do this. Antoine says he's in. The next night, Davey goes down to Dover and starts watching the planes land, to determine what his plan of action will be. Davey is arrested outside of the Air Force base. It turned out that Antoine was a DEA Agent. Davey is charged with conspiracy. What should be his defense?

There can be no "one person conspiracies." For this reason a single criminal and an undercover police officer cannot conspire. This alone is not sufficient under the MPC, however, which follows a "unilateral approach" to conspiracy.

In a jurisdiction in which assault is defined as "intentionally causing the victim to fear an immediate battery," I cut Keith off at a traffic light. He yells outside his car window: "next time I see you, I'm going to punch you in the face." A cop overhears this and arrests him for assault. What is Keith's best defense?

There was no threat to commit a present battery. A promise of future action is generally not an assault.

What is the origin of the "degrees" of murder?

These come from state murder statutes; not from the common law. Most jurisdictions distinguish between different degrees of murder based on criteria identified in the given state's murder statute.

What are the rules to determine when an intervening cause breaks the chain of causation sufficiently to constitute a superseding cause?

These rules differ according to whether the intervening force is dependent on or independent of the defendant's act. An intervening force that is a result of or response to the defendant's act is a dependent intervening cause. An independent intervening cause is one that would have occurred regardless of the defendant's act. A dependent intervening cause will supersede the defendant's act only when it is a totally abnormal response to the defendant's act. An independent intervening cause normally will supersede the defendant's act, except when the independent intervening force was foreseeable.

A burglary defendant breaks a window with a hammer, because he wants to enter that window to steal the hope diamond, which he knows is located in the building. Shortly after breaking the window, he sees that the door is open. He enters the door and starts looking for the diamond. Before he can find it, the cops arrest him. He is charged and tried for burglary. He raises the defense that there was no "breaking and entry," but rather a separate "breaking" and a separate "entry." Result?

This defense is valid. The breaking and entry must be related.

John tells his pit bull to charge at a visitor. The dog obliges and attacks the visitor. John is prosecuted for battery, but defends himself, arguing that he did not apply force to the victim, as is required by the law for a conviction for battery. What result?

This defense will fail. Where the defendant puts a force in motion, the force need not be applied directly by the defendant. John may be guilty of battery even though he did not personally touch the visitor.

A car nut sees a classic car rear ended by a taxi. He sees the drivers of both cars get out of their vehicles and exchange words. The car nut yells out "kick that cabbie's ass!" After yelling this exhortation, the car nut remembers that his father was a cabbie and was a hardworking, upstanding man for whom he had more affection than he could ever have for a car. The car nut begins to feel guilt, and yells: "Just kidding! Insurance will take care of everything! There is no reason to be mad at one another!" A cop then arrests the car nut for solicitation. At trial, the car nut argues that he withdrew from solicitation when he started encouraging the two motorists to make peace. Result?

This defense will not work. because the crime of solicitation is complete as soon as the solicitation is made, withdrawal cannot be a defense to the crime of solicitation. Even where the solicitor later changes his mind, the change of heart may be a defense to the underlying crime, but not to the solicitation crime.

I'm walking down the street in June. A person wearing a trenchcoat, hat and sunglasses walks up to a naif man and says "psst, wanna buy a gold Rolex watch? They're only $100!" Being naif, the customer doesn't understand that these are obviously stolen watches. He buys one and is later arrested for receiving stolen property. At trial, he argues that he didn't know that the property was stolen. Result?

This is a good defense. The defendant must either know or actually believe that the property is stolen to be guilty of receiving stolen property. An honest but unreasonable belief that the property is not stolen would likely prevent a conviction for receiving stolen property.

What is the "irresistible impulse" test?

This is the test some jurisdictions use to determine whether a defendant claiming insanity was insane. Under this test, a defendant will be found not guilty for reasons of insanity where he had a mental disease that kept him from controlling his conduct.

When talking first-degree murder, what does "premeditated" mean?

Thought-over beforehand. Can, however take place very quickly.

Tom and Vito are bank robbers in a jurisdiction which requires an overt act in furtherance of the conspiracy for conspiracy liability to attach. While chatting in a public park, Tom and Vito agree to rob a particular bank. Little do they know, two cops have been listening into their conversation. When the two men split, one cop follows Tom and the other follows Vito. Vito makes a beeline for the bank, which he scopes out, taking photos of each entrance and window. The cop who is following Vito arrests him, then calls his partner who arrests Tom. Both men are charged with conspiracy. In his defense at trial,, Tom argues that taking pictures of a bank was neither illegal nor substantial, and that--even if that activity had been somehow illegal or substantial--he didn't do it. Result?

Tom's defense will fail. The overt act required for comspiracy need not be criminal or unlawful, and need only be committed by one member of the conspiracy. Moreover, mere preparation will suffice.

When does the initial aggressor gain the right of self-defense?

Two ways in which this can happen: 1. Upon complete withdrawal perceived by the party who was initially attacked; 2. Escalation of force by the victim of the initial aggression.

A car nut sees a classic car rear ended by a taxi. He sees the drivers of both cars get out of their vehicles and exchange words. The car nut yells out "kick that cabbie's ass!" A cop arrests the car nut for solicitation. At trial, the car nut raises in his defense that he believed that the cab driver was intentionally ramming the classic car from behind. Result?

Under modern statutes specific intent defenses such as unreasonable mistake of fact are legitimate defenses to solicitation.

A drunken car nut sees a classic car rear ended by a taxi. He sees the drivers of both cars get out of their vehicles and exchange words. The car nut yells out "kick that cabbie's ass!" A cop arrests the car nut for solicitation. At trial, the car nut raises the defense of voluntary intoxication. Result?

Under modern statutes specific intent defenses such as voluntary intoxication are legitimate defenses to solicitation.

A suspect has ridden away the motorcycle of another, but leaves a note explaining that he needed to borrow the bike temporarily, to be able to file some court papers on time but would return it within fifteen minutes. After filing the papers, however, the suspect decides he loves the bike and decides to keep it. Could the leaving of the note be sufficient to preclude the suspect from being charged with larceny? What is the name of the doctrine governing this situation?

Under the "doctrine of continuing trespass," one who takes another's property without authorization, intending only to use it temporarily before restoring it unconditionally to its owner (i.e., one who is normally found not to have an intent to steal) may nevertheless be guilty of larceny if he later changes his mind and decides not to return the property after all. As a general rule, the initial taking must be "wrongful" (i.e., without the owner's authorization). If the initial taking was wrongful, the trespass is said to continue until the time the intent to steal is formed.

What is the "void-for-vagueness" doctrine?

Under the due process clause of the Fifth and Fourteenth Amendments of the Constitution, persons must be on notice that certain conduct is forbidden. Therefore, the USSC has required that criminal statutes be specific and give a person of ordinary intelligence "fair notice" of what conduct is prohibited. Furthermore, the void-for-vagueness doctrine requires statutes to be fair and consistent in their enforcement and not be arbitrarily or erratically enforced.

A bank robber robs a bank with a fake gun. As he's running out of the bank with the plastic gun, he sees a cop with weapon drawn upon him. The robber attempts to scare the cop into dropping his gun by pointing the plastic toy at the cop. The cop freaks out and squeezes off a few rounds, both of which land in an old lady's forehead, killing her instantly. Can the robber be charged with felony murder?

Under the old common law, all felons were liable for any homicide that occurred during the perpetration of the felony. the common law did not make exceptions for homicides committed by non-felons. Under the majority rule, there is no felony-murder liability when a non-felon kills a felon or a bystander. The majority position is the AGENCY THEORY, under which felony murder extends only when the killing is committed by one of the participating felons. Under the minority rule, felony murder includes killings by non-felons, such as a killing by a victim of a robbery, a bystander or the police. Even among jurisdictions following the minority rule, some will not impose felony-murder liability if the person killed was one of the felons (the Redline limitation), while others follow the common law approach of including all resulting deaths within felony-murder liability.

Describe the "misdemeanor-manslaughter rule."

Under this rule, an unintentional killing that occurs during the commission or attempted commission of a misdemeanor that is malum in se or of a felony that is not of the inherently dangerous type required for felony murder is classified as involuntary manslaughter. Note that the malum in se misdemeanor need not be independent of the cause of death, unlike felony murder.

Distinguish robbery and extortion.

Unlike robbery, extortion does not require threats of immediate or imminent physical harm, and property does not need to be taken from teh victim's person or presence.

When is use of lethal force in defense of a third-person lawful?

Use of lethal force in defense of a third person is lawful when necessary to defend a third person who is facing an immediate and wrongful deadly attack. As with self-defense, however, the amount of force used must be proportional to the initial attack. If something short of deadly force would effectively ward off the attack, deadly force is not necessary and is therefore not reasonable.

Must a victim resist or attempt escape for the defendant to be found guilty of false imprisonment?

Victims are not required to try to resist or attempt except where the defendant has the apparent ability to effectuate threats. HOWEVER, though victims need not affirmatively search for potential escape routes, a victim is not "confined" if he is aware of reasonable escape means.

Tom and Vito have been arrested as the sole conspirators of a two-man conspiracy. Tom is acquitted of all charges. What is the result for Vito, whose trial has not yet begun?

Vito should be acquitted. An acquittal of one co-conspirator in a two-person conspiracy traditionally results in the acquittal of the single remaining co-conspirator because at least two guilty parties are required for a conspiracy conviction. This alone is not sufficient under the MPC, however, which follows a "unilateral approach" to conspiracy.

When is voluntary or involuntary intoxication a defense to a crime?

When it negates the existence of an element of the crime.

When does insertion of a tool into the premises constitute an "entry" for the purposes of burglary?

When the insertion of the tool is to accomplish the felony (e.g., gun barrel is inserted in order to commit robbery within the home). Insertion for the purpose of gaining entry alone (such as picking a lock) is insufficient to constitute "entry."

When is mistake of fact a defense?

Where it negates the existence of a mental state required to establish a material element of the crime. In other words:where there would be no crime if the facts were such as the defendant thought them to be.

When does the defense of duress justify criminal conduct? When is the defense of duress NOT available?

Where teh defednant reasonably believes that the only way to avoid unlawful threats of great bodily harm or imminent death is to engage in unlawful conduct. For duress, the threat comes from human forces rather than from forces of nature. Duress, or coercion, is not available as a defense to murder.

Where does the defense of entrapment exist?

Where the criminal act is the product of creative activity originating with law enforcement officials and the defendant is in no way predisposed to commit the crime. Note that a defendant's past criminal record is relevant in proving predisposition.

What is the big split amongst M'Naghten jurisdictions?

Whether a defendant who is "laboring under...a defect of reason from a disease of the mind" is relieved from criminal responsibility if he believes that the act is either moral or legal. Some jurisdictions require--for the defendant to that the defendant's defect prevent him from knowing that the offense was wrong AND that the offense was immoral. Other jurisdictions will allow assertion of the defense if failure to comprehend immorality alone is shown. Yet other jurisdictions will allow assertion of the defense if failure to comprehend illegality is shown.

Describe the "Wharton Rule," and list the crimes affected by it.

With crimes that can only be committed by two or more people, there is no conspiracy unless the agreement involves at least one other person whose participation is not essential, by definition, to the perpetration of the crime. Crimes affected: bigamy, incest, gambling, giving/receiving bribes, adultery, dueling.

When will withdrawal limit a conspirator's liability under the conspiracy?

Withdrawal may cut off further liability for crimes committed in furtherance of the conspiracy if the withdrawing conspirator communicates his withdrawal to each of the co-conspirators. Under the MPC, withdrawal by a co-conspirator may be a valid affirmative defense to the charge of conspiracy itself where the renouncing party gives timely notice of his plans to all members of the conspiracy and performs an affirmative act to thwart the success of the conspiracy.

Can a perpetrator be guilty of both statutory and forcible rape?

Yes, but in most jurisdictions, he would be sentenced to the more serious crime of forcible rape because the less-serious crime of statutory rape would merge.

Abe takes a swing at Bob with a butcher knife in hand. Is Bob legally justified in shooting Abe with a gun?

Yes, if Bob cannot avoid the potentially-lethal harm through non-deadly force.

Can words alone be sufficient to constitute complicity?

Yes, one's words alone may be sufficient to make one an accomplice if the words assisted or encouraged the accomplice.

A state has a statute that makes sex with a minor under the age of 17 a strict liability crime. Joe meets Lolitaat a bar and assumes that she is 21 because 21 is the drinking age in that state. Lolita also tells Joe she is 21 and nothing about her appearance suggests that she may be lying. She also presents to Joe a completely convincing piece of false ID that states a birth date that would make her 21. In reality, Lolita is only 15. Joe has sex with her. Is he guilty of violating the statute?

Yes.

A suspect has ridden away the motorcycle of another, but leaves a note explaining that he needed to borrow the bike temporarily, to be able to file some court papers on time but would return it within fifteen minutes. Could the leaving of the note be sufficient to preclude the suspect from being charged with larceny?

Yes. If, at the time of the taking, the defendant intends to return the property to the victim unconditionally and within a reasonable time, there is no intent to permanently deprive, and therefore no larceny. The defendant must have the ability to return the property, even if something unanticipated stops the actual return of the property.

During a bank robbery attempt, the gunmen accidentally kill a bank teller with one of the guns that the gunmen are carrying when he drops his gun, causing it to discharge when it hits the floor. Can the getaway car driver be prosecuted for murder?

Yes. A felony murder is a killing proximately caused during the commission or attempted commission of a serious or inherently-dangerous felony. Felony murder includes both intentional and accidental killings, and includes liability for the acts of one's accomplices.

Jack discovers Diane cheating on him with another man. He chases off the Sancho, then leaves her in the bedroom and slams the door behind him, yelling "stay there until you're sorry. If you beg, I may let you out for breakfast." Though Jack intended to lock the bedroom door, he was so upset that he accidentally released the lock before slamming the door. Diane believed she was locked in and never checked the door. May Jack be guilty of false imprisonment?

Yes. Victims are not required to try to resist or attempt except where the defendant has the apparent ability to effectuate threats. HOWEVER, though victims need not affirmatively search for potential escape routes, a victim is not "confined" if he is aware of reasonable escape means.

A well-intentioned father's son is deployed in the military. The son has given the father the information that one would need to move and access the money in the son's bank accounts. The son gave his father such access so that, in the case of the son's death, incapacity or unavailability, the father would be able to pay the son's creditors. Suddenly, after years of stagnation, equities markets begin to show marked improvements. The father, a risk-loving investor, invests his son's money in high-risk tech stocks, all of which tank. Could the father be guilty of larceny?

Yes. "Intent to permanently deprive" may be shown by demonstrating that a defendant recklessly exposed the property to loss or dealt with the property in a manner involving a substantial risk of loss.

I break into my neighbor's garage at night to steal his motorcycle and succeed in stealing it. Can I be prosecuted for BOTH burglary and larceny?

Yes. A defendant who commits or attempts the intended felony can be convicted of both crimes.

A car nut sees a classic car rear ended by a taxi. He sees the drivers of both cars get out of their vehicles and exchange words. The car nut yells out "kick that jerkoff cabbie's ass!" The driver of the old car subsequently punches the cabbie. A cop arrests the car nut. The car nut is charged with battery. The car nut concedes that he is guilty of solicitation, but argues that battery is a step too far to describe his actions. Will the battery charge stick?

Yes. A solicitor is treated as an accessory before the fact and will thus be guilty of any solicited crime (attempted or completed) by the solicitee. The crime of solicitation, unlike conspiracy, merges with the target felony upon completion of the crime.

Defendant broke into and entered Victim's home at night. Defendant, a rival surfer, intended to break Victim's leg with a sledgehammer (a felony), so that Victim could not skate in a championship tournament. As Defendant attempted to bludgeon Victim's leg, Victim awoke, startled, and jumped out of bed, pouncing on Defendant. Defendant threw Victim off and Victim's head and neck hit the sharp corner of her four-post bed. Victim died from blood loss as a result of the injuries. Can Defendant be charged with murder?

Yes. Defendant is subject to prosecution for felony murder in a majority jurisdiction. A minority of states consider burglary with the intent to commit assault to be noncollateral, and the felony-murder rule is therefore inapplicable to that crime in those jurisdictions. A majority of states, however, find burglary with the intent to commit assault to be a collateral crime to which felony murder doctrine applies if a death results.

Can false imprisonment be accomplished without physical means? If not, why? If so, how?

Yes. False imprisonment may be accomplished by threatening to apply immediate physical force or invalidly asserting authority. The physical means include erecting physical barriers or applying physical force.

Ballsack is on his death bed and is hooked up to life support. I disconnect the life support, and he dies two hours earlier. His previous prognosis was for him to die in 24 hours. Were my actions the *actual* cause of his death for purposes of meeting the requirements of murder?

Yes. In situations when a victim is already dying, if the defendant's actions bring about the victim's death more quickly than if the defendant had not acted, the defendant's actions would be an actual cause of the killing.

For the purposes of meeting the elements of murder, can a defendant be found to have *actually* "caused" a victim's death if the victim was already dying, and the defendant's actions merely brought about the victim's death more quickly than if the defendant had not acted?

Yes. In situations when a victim is already dying, if the defendant's actions bring about the victim's death more quickly than if the defendant had not acted, the defendant's actions would be an actual cause of the killing.

Unknown to Paul, Killer intended to shoot and kill Fred with a high-powered rifle. Paul pushes Fred out of an airplane at 20,000 feet above ground. Just as Fred was about to hit the ground, Killer fired a bullet into his head, killing him instantly. Is Killer's act of shooting Fred the actual cause of Fred's death?

Yes. Killer did kill Fred. It doesn't matter that he killed him a second before he was about to die anyway.

I'm hanging out at a friend's house playing guitars with him. I notice that he has a '53 gold top Les Paul in the closet. As it fits perfectly in the case for my ****** '98 Epiphone LP, I swap the guitars. Since I'm in need of surgery in order to be able to work, I pawn the thing and get the surgery. I return to work. When I get my paycheck, I get the guitar out of hock and go to return it to my friend. On my way to the friend's house, I'm stopped by a cop for speeding. He notices the guitar in the back seat. I am charged with larceny. As a defense, I raise, in my defense, that I had only pawned the guitar, and was going to return it. Will I be successful?

Yes. Larceny defendants who pawn property may negate the "intent to permanently deprive" element by proof that, at the time of taking, they intended to redeem the property and return it to the victim.

A barber finds a wallet filled with cash laying on the coffee table in his shop. He opens it, and finds the driver license of a frequent customer, whose telephone number he has in the shop Rolodex. Without attempting to contact the customer who has lost his wallet, the barber takes the cash for himself, and throws the wallet itself in a Salvation Army collection box. Is the barber guilty of larceny?

Yes. Lost or mislaid property can be the subject of larceny, if the finder: 1. Intends to permanently deprive the owner of the property, and 2. Either -knows who the owner is, or -has reason to believe that he can find the owner's identity.

Does breaking into someone's tool shed with the intent to steal a tool satisfy the "dwelling house of another" element of burglary?

Yes. The "dwelling house of another" element of burglary encompasses structures within the curtilage of a home.

Can an owner be guilty of larceny of his own property?

Yes. The "property of another" element refers not to possession, rather than ownership if the owner deprives another of lawful possession of the owner's own property.

Defendant intentionally runs over Victim with Defendant's automobile, causing Victim to suffer serious (but not life-threatening) injuries. Victim is rushed to a hospital and given medical treatment. Physician negligently treats victim in a manner that is unfortunately common. Can defendant be found to have proximately caused Victim's death?

Yes. The doctor's negligence was a dependent intervening cause, and therefore did not supersede the Defendant's wrongdoing.

Abel finds Bob really annoying, and wants to piss him off, scare him, and cause him a little pain. Abel decides to go up to Bob, pull out a switchblade, and cut a line into his face. Little does Abel know, Bob is a hemopheliac with a heart condition. After Abel does his slicing job on Bob's face, Bob quickly and simultaneously has a heart attack and bleeds out. Bob dies. Was Abel's knife attack a proximate cause of Bob's death?

Yes. The tort maxim "you take the plaintiff as you find him" applies to crime victims as well. That is, if the harm results from a special sensitivity (such as hemophilia or other preexisting medical condition), the defendant's act is a proximate cause of the harm, regardless of whether the defendant could have foreseen the unique medical condition.

If a a cop uses deadly force against a person who he believes is a fleeing dangerous felon, and it later is found that the person was not a dangerous felon, or that no dangerous felony took place, can the cop use the defense of mistake?

Yes. Unlike private citizens, police officers who mistakenly use deadly force in such fleeing felon situations may be justified.

Defendant 1 stabs Victim in the heart with a knife. Simultaneous with the stabbing, Defendant 2 shoots Victim in the head. Medical testimony conclusively establishes that either the knife or bullet wound alone was sufficient cause to instantly kill Victim. Was Defendant 1's act the actual cause of the victim's death?

Yes. When there are multiple causes or other parties than the defendant responsible for the criminal result and DEFENDANT'S ACT WAS A SUBSTANTIAL FACTOR CAUSING THE CRIMINAL RESULT if the defendant's actions were the "actual cause" of a criminal result.

Can a defendant be guilty of murder though he did not foresee the exact chain of events that resulted in the victim's death?

Yes. Where the victim's death was a "natural and probable" consequence of the defendant's conduct, the defendant may be guilty of murder, even where he did not foresee the exact chain of events that resulted in the victim's death.

Defendant thought it would be funny to drive on the wrong side of the street "like they do in England." He did so, at high, speed, through a residential neighborhood, and veered into kids crossing the street at a crosswalk (who were looking the other way for oncoming traffic). Did defendant have the right mens rea for murder?

Yes; Defendant has the correct mens rea for depraved heart murder. The killing of the kids was an unintentional killing that resulted from conduct involving a wanton indifference to human life and a conscious disregard of an unreasonable risk of death or serious bodily injury. Moreover, the risk was not created for a socially-reasonable purpose.

Criminal battery is a ____________ crime.

general intent. Therefore, either recklessness or negligence will suffice.

Describe "intent to cause serious bodily harm" as that term is used to describe one of the means of satisfying the mens rea requirement of murder.

significant but non-fatal injury. Can arise from a conscious desire or a substantial certainty that the defendant's actions will result in the victim's injury.

Intent-to-kill murder is a ________ crime.

specific intent

The inchoate crime of solicitation is a ________ crime.

specific intent

Larceny is a ________ crime.

specific intent There must be intent to permanently deprive.

Define "taking" as that term relates to the crime of larceny.

the assertion of dominion and control over an article of property by a defendant who does not have lawful possession.

In a jurisdiction in which assault is defined as "intentionally causing the victim to fear an immediate battery," I run up to someone I hate with a bucket of mixed poop and piss. I threaten to throw it on them unless they give me money. I am arrested and charged with assault. I raise the defense that there was no physical injury threatened. What result?

the defense will afil. Any threatened contact, inculding offensive or insulting contact, is sufficient to constitute an "apprehension" assault. There need be no actual pain or physical injury threatened. Spitting on someone is an assault

Define "false imprisonment."

the intentional, unlawful confinement of one person by another.

What is required to satisfy the "breaking" element of common law burglary?

today, most burglary statutes have relaxed the breaking element to include even the slightest enlargement of an opening. Others have even deleted the breaking element altogether. This element may be satisfied where a defendant gains entry by fraud, deception, or threat of force (constructive breaking)


Related study sets

Finance 2000 chapter 1 questions

View Set

CM 2: Introduction à l'éthique en recherche

View Set